You are on page 1of 32

148 5: Le ga l a nd Ethica l Is s ue s in P sychia try a nd Me d ic ine

(A) Ad m inister a stat d ose of lorazep am flattered by the p atient’s sexu al interest and are su r-
2 m g IM and reevalu ate w hen the p rised by you r ow n interest in the patient.
p atient is m ore calm .
(B) Ad m it the patient involuntarily. 31. Which of the follow ing is the most appropriate
(C) Call Child Protective Services becau se cou rse of action?
the infant is at risk of harm . (A) Decline p articip ation in the relationship
(D) Discharge the p atient and inform becau se sex w ith a form er p sychiatric
the hu sband and m other to bring p atient is u nethical.
the patient back to the hosp ital if her (B) Engage in sexu al relations becau se sex
sym ptom s d o not im prove in the next w ith a cu rrent or form er p sychiatric
few w eeks. p atient is ethical.
(E) Give the p atient a 2-w eek sup ply of (C) Engage in sexu al relations becau se sex
flu oxetine to treat her d ep ression and w ith a form er p sychiatric p atient is
then arrange ou tpatient follow -up. p erm issible only if you d o not exp loit
you r p ast p osition of au thority.
30. A physician is at a comm unity fair w ith her (D) Engage in sexu al relations becau se there
spouse w hen a patient approaches the p sychi- is no established p rofessional cod e of
atrist to say hello. The spou se d oes not recog- ethics regard ing sex w ith p sychiatric
nize the patient. The patient d oes not introd u ce p atients.
him self to the sp ou se, nor d oes the physician
(E) Inform the p atient that the p rofessional
acquaint the tw o. After a brief conversation,
cod e of ethics requ ires that you w ait
the patient politely excu ses him self and leaves.
1 year after term ination before you can
On the w ay hom e, the spouse asks, “Who w as
ethically engage in sexu al relations.
that man you w ere talking to earlier?” The
physician should d o w hich of the follow ing?
32. Which of the follow ing w ou ld be the m ost
(A) Answ er the sp ou se’s qu estion tru thfu lly. ap p rop riate cou rse of action if the above
(B) Ask the sp ou se to gu ess the id entity of p atient w ere you r m ed ical or su rgical p atient?
the person. (A) Decline particip ation in the relationship
(C) Inform the sp ou se that he m u st first becau se sex w ith a cu rrent p atient is
p rom ise not to reveal the id entity of the u nethical.
p atient before answ ering the qu estion. (B) Engage in sexu al relations becau se sex
(D) Inform the sp ou se that revealing w ith a cu rrent nonp sychiatric p atient is
su ch inform ation w ou ld com prom ise ethical.
confid entiality. (C) Engage in sexu al relations becau se sex
(E) Lie to the sp ou se to p rotect the id entity w ith a cu rrent nonp sychiatric p atient is
of the patient. p erm issible only if you d o not exp loit
you r p ast p osition of au thority.
Questions 31 and 32 (D) Engage in sexu al relations becau se there
You have been treating a fem ale patient w ho has is no established p rofessional cod e of
been seeing you for p sychod ynam ic p sychotherap y ethics regard ing sex w ith nonp sychiatric
for app roxim ately 6 m onths. N ear the end of the p atients.
su ggested cou rse of treatm ent, the p atient reports (E) Inform the p atient that the p rofessional
that she feels m arked ly better abou t her progress cod e of ethics requires that you w ait
and attributes her im p rovem ent to you r exp ertise. 1 year after term ination before you can
Prior to the last session, she confesses that she has ethically engage in sexu al relations.
alw ays found you attractive and that she w ou ld like
begin an intim ate relationship w ith you . You feel 33. A 52-year-old man, for w hom you have been
the primary care physician for the last 20 years,
Que s tions : 30–36 149

w as recently diagnosed w ith amyotrophic (B) Ensu re that the p atient is established
lateral sclerosis. The disease has rapid ly pro- w ith a casew orker to su p ervise the
gressed and he has experienced multiple respi- p atient in the com m unity before
ratory complications that likely w ill require a d ischarge.
tracheotomy. Severe muscle w eakness and atro- (C) Establish follow -u p at the p atient’s
phy are apparent in all limbs. The patient states outpatient com m u nity m ental health
that there is no meaning in continuing w ithout facility p rior to d ischarge.
his physical capacities. H e asks for your help (D) Keep the p atient on the u nit as long as it
in ending his life in a humane and d ignified is m ed ically necessary.
manner. H is MSE is unremarkable and there
(E) Sp eak w ith the inp atient social w orker
is no evidence of any psychiatric d isorder. You
to d eterm ine w hether the patient is
d iscuss the patient’s request w ith his family and
eligible for a loan.
they unanimously support his d esire to “end
the suffering.” Which of the follow ing w ould
Questions 35 and 36
be the most appropriate course of action?
You receive a subp oena from an attorney rep resent-
(A) Ignore the p atient’s requ est.
ing a p arty that has filed a law su it against one of
(B) Provid e the p atient w ith inform ation
you r p atients. The su bp oena p ertains to releasing
regard ing how to effectively end his
the m ed ical record s of you r patient.
life.
(C) Provid e the p atient w ith enou gh 35. Which of the follow ing is the m ost ap p rop riate
m ed ication refills to p rovid e a lethal next step ?
d ose.
(D) Refu se to p articip ate in assisting the (A) Contact the attorney w ho obtained
p atient w ith su icid e and focu s on the su bpoena to d iscu ss the p rocess of
resp ond ing to the p atient’s end -of-life releasing the m ed ical inform ation.
issu es. (B) Do not release the inform ation and
(E) Resp ect the p atient’s w ishes by help ing contact your patient regard ing the
him end his life in a p ainless and su bpoena.
resp ectfu l m anner. (C) Release the m ed ical record s d irectly to
the presid ing jud ge.
34. You are an inp atient p sychiatrist treating a (D) Release the m ed ical record s u p on
p atient w ith bip olar I d isord er. The p atient receiving the su bp oena.
has a long history of m ed ication noncom p li- (E) Requ est that the p atient sign a release
ance resu lting in severe, persecu tory d elu - of inform ation form and tu rn over the
sions d u ring his m anic ep isod es. After 1 w eek record s.
of treatm ent, you receive a p hone call from
hospital ad m inistration inform ing you that 36. A cou rt hearing has been organized by you r
the p atient’s insu rance w ill not cover the cost p atient’s attorney to qu ash the su bpoena you
of an ad d itional inpatient stay. You are encou r- have been issu ed . At the hearing, the ju d ge
aged to d ischarge the p atient so that the hosp i- rules that you should release the entire m ed i-
tal w ill not have to incu r these costs. You feel, cal record even thou gh your patient has not
how ever, that the p atient requ ires m ore tim e consented to the release of information. Which
on the inpatient u nit because of safety con- of the follow ing w ould be the most appropriate
cerns. Which of the follow ing actions w ou ld next step?
be the m ost ap p rop riate?
(A) Ap p eal to the state’s Sup rem e Cou rt to
(A) Contact the insu rance com pany w ithou t block the release of the m ed ical record .
the patient’s perm ission and request (B) Refu se to d isclose the p atient’s m ed ical
coverage for ad d itional d ays. record regard less of the cou rt ord ers.
150 5: Le ga l a nd Ethica l Is s ue s in P sychia try a nd Me d ic ine

(C) Release only inform ation that w ill not (A) “Becau se you r d ep ression is im p roved ,
incrim inate you r p atient. it w ou ld be ap p rop riate for u s to be
(D) Release the com p lete m ed ical record to bu siness p artners.”
the jud ge. (B) “I can invest w ith you only w hen ou r
(E) Work ou t a p lan of legal action w ith treatm ent is nearing its end .”
you r p atient. (C) “I have to d ecline; it p otentially
m ay interfere w ith ou r treatm ent
37. A cou rt-ap pointed forensic psychiatrist is relationship .”
evalu ating a m an charged w ith arm ed robbery (D) “It is p robably a bad id ea; I’m alread y
to d eterm ine w hether he is com petent to stand com m itted in other investm ents.”
trial. Du ring the interview, the d efend ant con- (E) “Thank you for thinking abou t m e. I’d
fesses to m u rd ering a w om an 3 years ago be honored to invest w ith you .”
and hid ing her bod y in an u nd isclosed area.
Which of the follow ing actions w ou ld be the Questions 40 and 41
m ost appropriate?
A 23-year-old Cau casian Catholic w om an w ith a
(A) Avoid d etails of the d efend ant’s prior
history of m ajor d epressive d isord er and m u ltiple
crim inal history in the report.
m ed ical illnesses p resents to your office after argu -
(B) Encou rage the d efend ant to sp eak w ith ing w ith her hu sband . She is a stay-at-hom e m other
his attorney about the m u rd er. of fou r child ren and ad m its that she is u nable to
(C) Im m ed iately notify au thorities of the w ork becau se of her m ed ical lim itations. She rep orts
d efend ant’s confession. that her hu sband d oes not allow her to visit w ith
(D) Inclu d e this d etail in the final rep ort. friend s or fam ily w ithou t his consent and generally
(E) Withd raw from the case. d oes not help w ith child care or other household
d u ties and obligations. As a result, she is becom -
38. A 28-year-old w om an w ith m ajor d ep ressive ing increasingly resentfu l, d ep ressed , and isolative.
d isord er has been seeing you for w eekly psy- Du ring the session, she ad m its to thou ghts of killing
chotherap y and has failed to p ay her bill for herself. Of note, she has a p ast history of an over-
2 m onths. Which of the follow ing is the m ost d ose attem pt w ith aspirin.
ap p rop riate next step ?
40. Which of the follow ing is her strongest risk
(A) Contact the patient’s family to d etermine
factor for su icid e?
if the patient is financially stable.
(B) Contract w ith a billing collector to (A) Age
d em and im m ed iate p aym ent. (B) Gend er
(C) Inform the p atient that you w ill not see (C) Marital statu s
her if she d oesn’t pay for her treatm ent. (D) Past history of su icid e attem p t
(D) Inqu ire as to the reasons she has been (E) Term inal m ed ical illness
avoid ing p aym ent at the p atient’s next
visit. 41. Which one of the follow ing, if d ocu m ented ,
(E) Term inate the treatm ent. w ould mostly likely legally protect a physician
in the event of a patient suicid e?
39. A wealthy 46-year-old male banker is in psy-
(A) A w ritten “no self-harm ” contract
chotherapy with you for treatment of a single
signed by the p atient.
episode of major depression. After significant
improvement in his symptoms, he offers you the (B) An assessm ent of su icid e risk and
opportunity to take part in one of his financial p rotective factors.
ventures. The investment appears to be sound (C) The p atient’s m issed ap p ointm ents.
and fairly lucrative. Which of the following is (D) The p atient’s refu sal to consid er
the most appropriate response to the banker? p harm acological treatm ents.
Que s tions : 37–45 151

(E) The patient’s verbal p rom ise to seek ative on the u nit. H e becom es intru sive, not able to
m ed ical attention if feeling su icid al. be red irected , and d em and s im m ed iate release. You
exp lain to him that you feel he is gravely d isabled
42. A 36-year-old m an w ith a history of bip olar I and u nable to care for him self. H e d isagrees w ith
d isord er is brought to the em ergency d epart- you and d em and s “d u e p rocess.”
m ent by p olice after stabbing a patron in a
bar room braw l. H is blood alcohol level w as 44. On w hich of the follow ing legal princip les is
0.320 u pon arrival, and the p atient requ ired the patient’s request for a hearing based ?
intram u scu lar (IM) H ald ol for agitation. The (A) Actu s reu s
p atient has no recollection of the event, and
(B) H abeas corp us
the victim d ied 2 hours later. The p atient has a
(C) Mens rea
history of assau lt w hile being m anic. H e cu r-
rently p articip ates in w eekly p sychotherap y (D) Parens p atriae
sessions and attend s his m ed ication m anage- (E) Rights u nd er the Fou rth Am end m ent
m ent ap pointm ents every m onth. H is law yer
has chosen to assert an insanity p lea in d efense 45. The cou rt agrees that the p atient is severely
of the patient. Which of the follow ing factors d isabled and in need of acu te m ed ical m anage-
is m ost likely to u nd erm ine his assertion of the m ent. You initiate treatm ent w ith qu etiapine
insanity d efense? and his sym ptom s of m ania m arked ly d im in-
ish. The p atient requ ests im m ed iate d ischarge
(A) Inability to recall the event. and agrees to follow u p w ith a p artial hospi-
(B) Mental d isease or d efect. talization program . You feel that he w ou ld
(C) Prior history of assau lt. benefit from fu rther inp atient treatm ent bu t is
(D) Voluntary intoxication. no longer gravely d isabled or a threat for self-
(E) Violent nature of the crim e. harm . Which of the follow ing is the next m ost
ap p rop riate step ?
43. You are a forensic p sychiatrist hired as an
(A) Ap pease the patient by increasing
exp ert w itness by the d efense attorney of a
sm oking privileges.
m entally ill p atient charged w ith crim inal m is-
(B) File for another cou rt hearing to d etain
cond u ct. The attorney is seeking you r help in
the patient fu rther.
convincing the ju ry that the patient w as m en-
tally ill at the tim e of the crim e. Which of the (C) Ignore the p atient’s requ est becau se he
follow ing is your prim ary resp onsibility as an has been com m itted by the court.
exp ert w itness? (D) Persu ad e the patient to stay for a few
m ore d ays.
(A) Cou ntering evid ence of crim inal
(E) Release the patient to the partial
resp onsibility.
hosp italization p rogram .
(B) Establishing reasonable d ou bt.
(C) Evalu ation, d iagnosis, and initiation of Questions 46 and 47
treatm ent of the accu sed .
(D) Obtaining a not guilty by reason of You are consu lted by the m ed ical team to evalu -
insanity verd ict. ate a w om an on the m ed ical u nit w ho su ffers from
a m ajor neurocognitive d isord er (d em entia). The
(E) Rend ering an opinion based on
internist believes that she requ ires a central line for
reasonable m ed ical certainty.
flu id s and m ed ication, bu t is u nsu re if she is able to
Questions 44 and 45 fu lly com p rehend the risks and benefits of the p ro-
ced ure. The team is requ esting help in d eterm ining
A 48-year-old m an is involu ntarily ad m itted to the her capacity to give inform ed consent.
hosp ital after an acute m anic episod e. The patient
is hyperactive, d em and ing, and increasingly talk-
152 5: Le ga l a nd Ethica l Is s ue s in P sychia try a nd Me d ic ine

46. Which of the follow ing com ponents is the d enies any hallu cinations or d elu sions. Which
m ost im portant in obtaining inform ed consent of the following items from your evaluation of
in this case? the patient most raises your suspicion regarding
a d iagnosis of m alingering?
(A) Ability to read and w rite.
(B) Absence of m ental illness. (A) Du ration of sym p tom s.
(C) Involving fam ily m em bers in this (B) Flashbacks w hile aw ake.
d iscu ssion. (C) H istory of m ajor d ep ressive d isord er.
(D) Petitioning a court to establish the (D) Lack of physical inju ries.
p atient’s com petence. (E) Prior incarcerations.
(E) Raising alternative treatm ent op tions.
49. One of you r p atients consistently m isses
47. The fam ily is su bsequ ently contacted abou t ap p ointm ents w ithou t giving you ad vance
her cond ition, and they requ est a m eeting notice. After num erou s failed attem p ts at
w ith the treatm ent team . Du ring the m eeting, resolving this issu e w ith the patient, you real-
the patient’s eld est son notifies you that the ize that the p atient’s behavior is not changing
p atient d id create a living w ill ap proxim ately and you d ecid e to d ischarge the p atient from
1 year ago, but they are unsure whether it would you r care. Which of the follow ing m ost ap p ro-
be useful. Which of the follow ing w ould you p riately d escribes w hat you shou ld d o?
tell them best d escribes the pu rpose of a living
(A) Contact the patient’s fam ily/ friend s
w ill?
to requ est their help in im p roving the
(A) Absolves personal resp onsibility. p atient’s attend ance.
(B) Arranges for fu neral services and (B) N otify the p atient’s insu rance com p any
d istribu ting her estate. that she is d ischarged from you r care.
(C) Establishes p ersonal p references (C) N otify you r staff that the p atient is not
regard ing end -of-life issu es. to be given fu rther appointm ents.
(D) Prevents the p atient from changing her (D) Write a letter to the p atient notifying her
m ind about life su pport if she becom es that she has been d ischarged effective
term inally ill. im m ed iately d u e to her failu re to
(E) Requ ests p hysician-assisted su icid e if com ply w ith treatm ent.
she becom es term inally ill. (E) Write a letter to the p atient stating that
she w ill be d ischarged in 60 d ays d ue to
48. A 28-year-old m an involved in a m otor vehicle her failu re to com p ly w ith treatm ent.
accid ent brings a law su it against the d river.
Em ergency d ep artm ent record s d o not show 50. You are asked by the cou rt to p rovid e a foren-
any p hysical inju ries, but the p atient is claim - sic evaluation for a child cu stod y case. The
ing to su ffer from p osttrau m atic stress d is- child had been living w ith his biological
ord er (PTSD). You are asked to evalu ate the m other and stepfather since birth. Recently,
p atient’s sym p tom s. H e com p lains of d istress- the child ’s stepfather passed aw ay from lu ng
ing d ream s of the accid ent, having flashbacks cancer leaving the u nem ployed m other alone
w hile aw ake, and avoid ing the intersection to raise the child . The child ’s biological father,
w here the accid ent occu rred . H is sym p tom s w ho is a renow ned orthop ed ic su rgeon in the
have lasted for 3 m onths. Past psychiatric his- com m unity, is requ esting full cu stod y becau se
tory is significant for m ajor d epressive d isor- he asserts that he is better able to financially
d er, im p u lsivity, and violent behavior. H e has su pport the child . The child ’s m other refu ses
been incarcerated nu m erou s tim es, show ing to relinquish cu stod y d u e to concerns that
a blatant d isregard for the law. On MSE, the leaving her hou sehold w ill em otionally im p act
p atient is likable and coop erative. H is m ood is the child . She also im p lies that the father has
rep orted as being d ep ressed and he cu rrently “anger p roblem s” and voices su sp icion as to
Que s tions : 46–51 153

w hether he is, in fact, the biological father. that your colleagu e has been m aking increas-
Which of the follow ing factors w ou ld be the ingly careless m istakes and m issing im p ortant
m ost im p ortant in d eterm ining w hich p arent m eetings and d iscu ssions. Occasionally, you
shou ld get custod y? observe that his hand s shake w hen he is d ocu -
m enting his notes or hold ing his charts. You
(A) Who can p rovid e for the best interests
are concerned that he m ay have an alcohol use
of the child ?
d isord er. Which of the follow ing is the m ost
(B) Who has the highest level of ed u cation? ap p rop riate cou rse of action?
(C) Who is in the best m ental and p hysical
health? (A) Confront the colleagu e and d em and that
he seek ad d iction cou nseling services.
(D) Who is the m ost financially stable?
(B) Contact the Dru g Enforcem ent
(E) Who is the biological p arent?
Ad m inistration to rescind his license.
51. You are an internal m ed icine resid ent w ho has (C) Do nothing so as to avoid p ersonal
been assigned to rotate throu gh the inp atient liability.
m ed ical service for the next 2 m onths. Shortly (D) N otify the hosp ital’s com m ittee for
after you begin, you notice that one of you r im p aired p hysicians.
colleagu es consistently com es to w ork late and (E) N otify the local p olice.
sm ells of alcohol. Som etim e later, you notice
CHAPTER 6

Diffe re ntial Diag no s is and


Manag e me nt
Que s tions

D IRECTION S (Questions 1 through 60): For each of (C) H lop erid ol (H ld ol) nd p erp hen zine
the multiple-choice questions in this section, select (Tril fon)
the lettered answ er that is the one best response in (D) Mirt z p ine (Re eron) nd cit lop r
each case. (Celex )
(E) Zip r sid one (Geod on) nd sertr line
Questions 1 and 2 (Zoloft)

A 43-ye r-old w o n p resents to you r office tell- Questions 3 and 4


ing you th t recently she h s been exp eriencing n
incre se in the volu e of voice th t she h s been A 29-ye r-old w o n w ho ju st d elivered 3 w eeks
he ring for ye rs. It const ntly criticizes her beh v- go is referred bec u se her obstetrici n noticed th t
iors reg rd less of her ood . She lso notes th t for she ppe red to be d isheveled . Upon initi l inter-
ost of the p st ye r, her ood h s been very low . view , the p tient tells you th t she h s been feel-
She is no longer ble to get ny ple su re fro w tch- ing d ow n since d elivering her son. She tells you
ing television. H er sleep is p oor nd her energy is th t w hile she continu es to c re for her son, she is
low . In d d ition, she d escribes 20-lb w eight loss in grow ing incre singly d ep ressed bec u se she d oes
the p st ye r bec u se she no longer feels the need to not get ny p le sure fro t king c re of hi . She is
e t. She d enies the u se of ny d ru gs or lcohol. u n ble to sleep nd h s not been e ting uch either.
She feels tired ll the ti e. She lso tells you th t t
1. Which of the follow ing d i gnoses best ti es, she w ill he r b by in the b ckground cry-
ccou nts for this p tient’s sy p to s? ing, bu t w hen she checks on her son, he is sleep ing
sound ly. She d enies ny thoughts of w nting to hu rt
(A) Bip ol r II d isord er her son or herself.
(B) M jor d epression w ith psychotic fe tures
(C) Schizo ffective d isord er 3. Which of the follow ing d i gnoses is the ost
(D) Schizop hreni likely?
(E) Schizoid p erson lity d isord er (A) Ad ju st ent d isord er
(B) Brief p sychotic d isord er
2. Which of the follow ing ed ic tion co bin -
(C) M jor d ep ressive d isord er w ith
tions w ou ld best tre t this p tient’s sy pto s?
p erip rtu onset nd p sychotic fe tu res
(A) Div lp roex sod iu nd lor zep (D) Postp rtu obsessive-co p u lsive
(Ativ n) d isord er
(B) Flu oxetine (Proz c) nd d i zep (E) Schizo ffective d isord er, d ep ressed typ e
(V liu )

165
166 6: Diffe re ntia l Diag nos is a nd Ma na g e me nt

4. Which of the follow ing is the ost pp rop ri- Questions 7 and 8
te first step in her tre t ent?
A 62-ye r-old w o n p resents to you r office long
(A) Flu oxetine (Proz c) nd qu eti p ine w ith her d u ghter. H er d u ghter exp l ins to you
(Seroqu el) th t l tely, her other h s been extre ely con-
(B) H lop erid ol (H ld ol) cerned bout her bod y od or. Despite ultiple re s-
(C) H osp it lize the p tient su r nces fro her d ughter th t she d oes not s ell,
(D) Lithiu she continues to be concerned th t other people find
her bod y od or extre ely offensive. The p tient tells
(E) Su p p ortive ther p y
you th t she know s th t she s ells bec u se she c n
s ell her ow n bod y od or ll the ti e d espite t king
Questions 5 and 6
u ltip le show ers throu ghou t the d y. Despite being
A 64-ye r-old w o n is brou ght to the e ergency concerned bout her od or, she continues to w ork
d ep rt ent by her neighbor, w ho s ys “ y friend fro ho e s she h s d one for the p st 30 ye rs
isn’t cting right.” The p tient requ ires the su p p ort nd she continu es to sp e k w ith friend s on the tele-
of nurse w hile w lking to n ex in tion t ble. p hone. She p ys her onthly bills w ithou t the help
Ex in tion reve ls th t she c nnot correctly id en- of her f ily. H er thinking otherw ise see s logic l
tify the se son or the tow n she is in. She d oes not nd pp rop ri te.
recognize her neighbor. She is in ttentive nd see -
ingly p thetic to the ctivity rou nd her. She d ozes 7. Which of the follow ing is the ost likely
off repe ted ly d uring the interview , bu t e ch ti e d i gnosis?
is rou s ble nd resu es nsw ering qu estions. H er (A) Delu sion l d isord er
nsw ers re illogic l nd inconsistent.
(B) M jor d ep ressive d isord er w ith
Vit l signs re w ithin nor l li its nd she
p sychotic fe tu res
is neither tre u lou s nor d i p horetic. N eu rologic
ex in tion find s bil ter l sixth nerve p lsy nd (C) Obsessive-co p u lsive d isord er
horizont l nyst g u s. Urine toxicology screen nd (D) P r noid p erson lity d isord er
blood lcohol level re neg tive. (E) Schizop hreni

5. Which of the follow ing is the ost likely 8. Which of the follow ing w ou ld be the ost
d i gnosis? p p rop ri te tre t ent?
(A) Acu te su bd u r l he to (A) C rb zep ine (Tegretol)
(B) Alcohol w ithd r w l (B) Electroconvu lsive ther p y (ECT)
(C) Folic cid d eficiency (C) Flu oxetine (Proz c)
(D) N or l p ressu re hyd rocep h lu s (N PH ) (D) Lithiu (Esk lith)
(E) Wernicke encep h lop thy (E) Ol nz p ine (Zyp rex )

6. Which of the follow ing is the ost i p ort nt Questions 9 and 10


first step in n ging this p tient?
A 29-ye r-old n w ith history of chronic schizo-
(A) Ad inistr tion of benzod i zep ine p hreni co es to the e ergency d ep rt ent w ith
(B) Ad inistr tion of folic cid te p er tu re of 102.9°F, l bile blood p ressu re rising
(C) Ad inistr tion of thi ine to 210/ 110 H g, p u lse of 110/ in, nd resp i-
(D) Co p u ted to ogr p hy (CT) sc n of the r tory r te of 22 bre ths/ in. This p tient’s ed ic -
he d tions includ e h lop erid ol, benztrop ine (Cogentin),
nd clon zep (Klonop in). H e c nnot correctly
(E) Intr venou s (IV) flu id s nd observ tion
id entify the d y, d te, or ye r, nd believes hi self
to be in city fro w hich he oved 10 ye rs go.
A f ily e ber ind ic tes th t 3 d ys go he w s
Que s tions : 4–13 167

he lthy nd co pletely oriented nd th t he h s no 11. Which of the follow ing shou ld be the ost
signific nt ed ic l or surgic l history. i ed i te n ge ent?
Physic l ex in tion reve ls th t he is in cute
(A) Ad ission to ed ic l u nit
d istress w ith hypertonicity. L bor tory ex in tion
reve ls cre tinine phosphokin se (CPK) of 45,000 (B) Ad ission to p sychi tric u nit
IU/ L, w hite blood cell cou nt of 15,000/ µL nd no (C) Med ic tion nd d isch rge to close
left shift, sod iu of 145 Eq/ L, nd cre tinine of f ily e ber
2.5 g/ d L. Lu b r p u nctu re prod u ces cle r fluid (D) Referr l to n ou tp tient p sychi trist
w ith slightly elev ted p rotein cou nt. (E) Restr ints nd ed ic tion in the
e ergency d ep rt ent
9. Which of the follow ing is the ost likely
d i gnosis? 12. Which of the follow ing ed ic tion(s) w ou ld
(A) Anticholinergic synd ro e be the ost p p rop ri te?
(B) Centr l nervou s syste (CN S) infection (A) Benzod i zep ine only
(C) M lign nt hyp erther i (B) Lithiu nd selective serotonin
(D) N eu rolep tic lign nt synd ro e (N MS) reu p t ke inhibitor (SSRI)
(E) Prolonged i obiliz tion (C) Lithiu only
(D) SSRI nd n ntip sychotic
10. With ppropri te tre t ent, the p tient recov- (E) Tricyclic ntid ep ress nt (TCA) only
ers co pletely nd returns ho e. In onth’s
ti e, he co es to the e ergency dep rt ent Questions 13 and 14
st ting th t the “voices in the w lls” re telling
hi to kill hi self. He h s t ken no edic tions A 22-ye r-old n is brou ght to the e ergency
since he left the hospit l. His vit l signs re st - d ep rt ent by p olice fter n ep isod e in w hich he
ble nd edic l workup is neg tive. Which of r ns cked the office w here he w orks looking for
the following ther pies should be initi ted first? “evid ence.” H e st rted this job 2 onths go fter
gr d u ting fro college. H e lives w ith fou r roo -
(A) ECT tes, nd he believes they re je lous of hi
(B) H lop erid ol d ep ot injections bec u se of his job nd h ve therefore been p oison-
(C) Ol nz p ine ing his food . H is f ily reve ls th t once before
(D) Physic l restr ints w hen he beg n college he w ent throu gh p eriod
(E) S fety onitoring only of “ cting cr zy” but got better w ithout tre t ent
nd h s d one w ell since. In the e ergency d ep rt-
Questions 11 and 12 ent, he is shou ting th t he h s been u p for w eek
w riting “cl ssic” book bou t ccou nting bu t th t
A 39-ye r-old w o n co es to the e ergency so eone t the office stole it fro hi . H e need s
d ep rt ent nd co pl ins th t since her boyfriend to be p hysic lly restr ined by e ergency d ep rt-
broke u p w ith her 3 onths go, she h s been sleep - ent secu rity. Physic l ex in tion nd co plete
ing nd e ting p oorly, h s lost ll interest in her l bor tory w orku p nd toxicology screen p rove to
w ork, nd feels gu ilty th t she d rove her boyfriend be neg tive.
w y. In the p st onth, she h s begu n to feel hop e-
less, help less, nd th t “life y not be w orth it.” In 13. Which of the follow ing ed ic tion(s) w ould
the p st 2 w eeks, she h s d eveloped belief th t be the ost p p rop ri te to initi te?
r re d ise se is rotting her he rt, nd over the p st
(A) Antip sychotic nd benzod i zep ine
w eek, voice h s been telling her she is no good nd
should t ke n overd ose of he rt ed ic tion she is (B) Bu sp irone (Bu Sp r)
prescribed . At first she w s ble to ignore the voice, (C) C rb zep ine (Tegretol)
how ever, she is now t the point th t she believes (D) Lithiu
she shou ld ct on it. (E) SSRI
168 6: Diffe re ntia l Diag nos is a nd Ma na g e me nt

14. Three onths l ter, the p tient sees his d octor (A) Acu te stress d isord er
for follow -u p . H e is t king lithiu nd h lo- (B) Ad ju st ent d isord er
p erid ol. H e is d oing w ell, except he co pl ins (C) MDD
of p inful u scle cr p ing. H is lithiu level
(D) P nic d isord er
is 0.8 Eq/ L. Which of the follow ing w ou ld
be the ost p p rop ri te next step in his (E) PTSD
n ge ent?
17. A p tient w ith history of bip ol r d isord er is
(A) Decre se the h lop erid ol d ose. d itted to p sychi tric hosp it l in n cute
(B) Decre se the lithiu d ose. nic ep isod e. H er ed ic tions inclu d e n
(C) Incre se the h lop erid ol d ose. SSRI nd benzod i zepine, w hich re both
(D) Incre se the lithiu d ose. d iscontinu ed on d ission. An ntipsychotic
nd ood st bilizer re st rted . Tw o d ys
(E) St rt b clofen (Liores l).
fter d ission, she c lls the nu rsing st ff to
her bed . She is extre ely frightened nd co -
Questions 15 and 16
p l ins excited ly th t she c nnot stop looking
A 29-ye r-old w o n tells her d octor th t bou t u p . On ex in tion, her eyes re noted to be
3 w eeks go she w s c rin g for ch ild w ho r n d evi ted up w rd , bil ter lly. Which of the fol-
into the street nd w s killed by bu s. Since then, low ing sid e effects is ost consistent w ith her
she c nn ot get th e i ge of the ccid ent ou t of h er p resent tion?
ind . Even in sleep , she d re s bou t it nd it p re- (A) N MS
vents her fro sleep ing ore th n few hou rs t
(B) Ocu logyric crisis
nigh t. She u sed to t ke bu s to w ork, bu t she n ow
d rives bec u se she c nnot be r to be ne r bu ses s (C) Retrocollis
this c u ses her to think bou t the ccid ent. In the (D) Torticollis
p st w eek, sh e h s begu n issin g w ork bec u se (E) Tris u s
she is u n co fort ble le ving her h ou se. Sh e feels
extre ely gu ilty, believing the ccid ent w s h er Questions 18 and 19
f u lt.
A 59-ye r-old w o n w ith long history of gener-
15. Which of the follow ing is the ost likely lized nxiety d isord er (GAD) tells her p ri ry c re
d i gnosis? d octor th t 2 d ys go w hile in crow d ed su p er-
rket she felt d izzy, long w ith ssoci ted he rt
(A) Acu te stress d isord er p lp it tions, p ressu re on her chest, nd fright-
(B) Ad ju st ent d isord er ening sense of d oo . Shortly there fter, she fell
(C) M jor d ep ressive d isord er (MDD) u nconsciou s nd w oke u p inu tes l ter to crow d
(D) P nic d isord er nd gor p hobi rou nd her. She felt so ew h t better nd rejected
others’ d vice th t n bu l nce be c lled . She
(E) Posttr u tic stress d isord er (PTSD)
qu ickly d e her w y ho e.
16. The p tient d ecid es g inst ny ed ic tion
18. Which of the follow ing w ou ld be the ost
bu t follow s u p w ith p sychother p y. A ye r
p p rop ri te next step in her n ge ent?
l ter, lthou gh she is no longer h ving d is-
tressfu l sy pto s rel ting to the ccid ent, she (A) Cognitive-beh vior l ther p y
feels s d nd te rfu l ost of the ti e, is h ving (B) Electroc rd iogr (ECG)
trou ble e ting, h s lost interest in g rd ening, (C) Re ssu r nce th t her cond ition is benign
nd w kes u p t 4 a m every orning, u n ble
(D) Short- cting benzod i zep ines
to get b ck to sleep. She const ntly feels tired
throughout the d y. Which of the follow ing is (E) SSRI
the ost likely d i gnosis?
Que s tions : 14–24 169

19. Which of the follow ing d i gnoses is the ost to feel d epressed , helpless, nd hopeless, nd
i p ort nt to ru le ou t first? he su bsequ ently overd oses on his ed ic tion.
H is w ife find s hi u nconsciou s nd p roceed s
(A) Acu te stress d isord er
to c ll 911. Which of the follow ing w ou ld be
(B) C rd iov scu l r d ise se the ost likely c u se of d e th?
(C) GAD
(A) C rd i c rrhyth i
(D) Illness nxiety d isord er
(B) Resp ir tory f ilu re
(E) P nic tt ck
(C) Seizu re
20. A 28-ye r-old le p resents to you r office (D) Shock
bec u se he w nts to stop p rticu l rly d is- (E) Stroke
turbing beh vior. H e tells you th t he often
goes on p u blic tr ins w ith the intention of try- Questions 23 and 24
ing to ru b his genit ls on other p eop le w ithou t
their consent. H e continu es to h ve f nt sies A 19-ye r-old n w ith no p reviou s psychi tric his-
bou t this, bu t he w ishes to stop bec u se he tory is noted by his college roo te to be cting
is fr id of getting into trou ble. Which of the biz rrely for the p st onth nd h lf, h ving con-
follow ing is his ost likely d i gnosis? vers tions w ith people w ho re not there, w lking
rou nd the d or itory roo n ked , nd ccusing the
(A) Exhibitionistic d isord er roo te of c lling the Dep rt ent of H o el nd
(B) Frotteu ristic d isord er Secu rity to h ve hi onitored . The p tient’s vit l
(C) Sexu l sochis d isord er signs re ll w ithin nor l li its nd his neurologic
(D) Sexu l s d is d isord er ex in tion show s no d eficits or bnor lities.
(E) Voyeu ristic d isord er
23. Which of the follow ing tests w ou ld be ost
usefu l in the initi l d i gnosis of this p tient?
Questions 21 and 22
(A) Co p lete blood cou nt (CBC)
A 53-ye r-old n presents to your office co pl in-
(B) Erythrocyte sed i ent tion r te (ESR)
ing of w orsening d epression. H e st tes th t he no lon-
ger enjoys spend ing ti e w ith his f ily, he is only (C) Liver fu nction tests
getting few hours of sleep t night, nd his ppetite (D) N oncontr st CT sc n of the br in
is uch low er th n it used to be. H e feels tired ll d y (E) Toxicology screen
long. You d ecid e to initi te n SSRI.
24. If the bove test w ere neg tive, w hich of the
21. Which of the follow ing sid e effects is he follow ing w ou ld be the ost likely d i gnosis?
ost likely to exp erience fter 3 onths of
(A) Delu sion l d isord er
tre t ent?
(B) M jor d ep ression w ith p sychotic
(A) Di rrhe fe tu res
(B) H e d ches (C) Schizop hreni
(C) N u se (D) Schizop hrenifor d isord er
(D) Sed tion (E) Su bst nce-ind u ced p sychosis
(E) Sexu l d ysfu nction
Questions 25 and 26
22. The p tient retu rns few ore ti es, nd
d u ring e ch visit, you incre se the d os ge of You h ve been sked by the su rgery te to ev lu -
the SSRI. H ow ever, he d oes not report ny te 35-ye r-old n w ho h d surgery to rep ir his
d ecre se in his d epressive sy pto s. Bec use fr ctu red right w rist 24 hou rs go nd is now co -
of his l ck of response to n SSRI, you d ecid e to pl ining of nxiety. The p tient h s been in the hos-
initi te tricyclic ntid ep ress nt. H e continu es pit l for 2 d ys. H is he rt r te is 120 be ts/ in, nd
Que s tions : 25–35 171

31. Which of the follow ing w ould be the ost Questions 34 and 35
d ngerou s sid e effect of bru p tly stop p ing the
bove ed ic tion? A 34-ye r-old fe le p resents to you r office s she
h s been feeling d epressed nd ngry for the p st
(A) Au tono ic hyp er ctivity cou ple of d ys bec u se she believes th t her boy-
(B) Seizu res friend is going to le ve her. As resu lt, she h s been
(C) H llu cin tions c lling hi every hour ju st to confir th t he is not
(D) Worsening nxiety le ving her. She tells you th t she h s h d ny rel -
tionships in the p st w hich h ve lw ys been very
(E) Vo iting
rocky, c u sing her e otions to const ntly go u p nd
d ow n. When ny of these rel tionships end ed ,
Questions 32 and 33
she cu t on her r s sup erfici lly to “relieve stress.”
A 73-ye r-old n is d itted to the hosp it l for At this ti e, she tells you th t she d oes h ve so e
co u nity- cqu ired pneu oni nd d ehyd r - thou ghts of w nting to cut herself, bu t no thoughts
tion. On d y 2 of his hosp it liz tion, you re sked of w nting to end her life.
to ev lu te the p tient for d ep ression; the st ff h s
noted th t he see s very w ithd r w n. H e is not e t- 34. Which of the follow ing is the ost likely
ing or sleeping w ell. The nu rsing st ff reports th t d i gnosis?
l st night he w s ngry nd requ ested to le ve the (A) Bip ol r d isord er
hospit l. You t lk w ith the p tient’s f ily nd find
(B) Bord erline p erson lity d isord er (PD)
th t the p tient h s no previou s psychi tric history.
Prior to the onset of this illness 5 d ys go, he h d (C) H istrionic PD
no d ep ressive sy pto s nd no d ifficu lties w ith (D) M jor d ep ressive d isord er
cognition. On ex in tion, his vit l signs re te - (E) N rcissistic p erson lity d isord er
per tu re of 98.2°F, he rt r te of 87 be ts/ in,
blood pressure of 130/ 86 H g, nd p erip her l 35. Which of the follow ing is the ost pp rop ri-
oxygen s tur tion of 95% on roo ir. H e is d row sy te tre t ent for this d i gnosis?
nd oriented only to p erson. H is Mini-Ment l St te
(A) Arip ip r zole (Abilify)
Ex in tion (MMSE) score is 23/ 30.
(B) Cit lop r (Celex )
32. Which of the follow ing is the ost likely (C) Cognitive-beh vior l ther p y
d i gnosis? (D) Di lectic l beh vior l ther p y
(E) Psycho n lysis
(A) Anxiety d isord er
(B) Deliriu Questions 36 and 37
(C) F ctitiou s d isord er
(D) M jor d ep ressive d isord er (MDD) A 65-ye r-old w o n w ith p st ed ic l history
of non–insu lin-d ep end ent d i betes ellitu s nd
(E) M jor neu rocognitive d isord er
d epression is d itted w ith incre singly d epressed
(d e enti )
ood over the l st onth. She is u n ble to co -
plete her crossw ord pu zzles bec use of d ifficu lty
33. The ed ic l te sks for ed ic tion reco -
concentr ting. She h s trouble f lling sleep nd
end tions if the p tient beco es git ted .
lso w kes u p in the id d le of the night. She d enies
Which of the follow ing w ou ld be the ost
su icid l id e tion, but d oes feel gu ilty th t she is
pp rop ri te ed ic tion to reco end ?
d epressed . Prior to this episod e, she w s d oing w ell
(A) Benztrop ine nd w s ctively eng ged in co u nity volunteer
(B) Dip henhyd r ine grou p s. In the l st onth, she h s lost 13 lb d ue to
(C) Lor zep poor int ke. When sked w hy she is not e ting, she
st tes she is w orried th t she w ill beco e infected
(D) H lop erid ol
w ith b cteri . She h s been to her p ri ry p hysici n
(E) Thiorid zine
for n ev lu tion, bu t she cl i s everything w s
170 6: Diffe re ntia l Dia g nos is a nd Ma na g e me nt

his blood pressure is 160/ 106 H g. H e is febrile 28. Which of the follow ing w ou ld be the ost
nd reports th t he h s never su ffered nything like pp rop ri te tre t ent for this p tient?
this before. H e is not in ny p in nd h s no pre-
(A) A low -sti u lu s environ ent
viou s psychi tric history. H is ed ic tion inclu d es
only cet inophen for p in control. You note th t (B) Benztrop ine
he is d i phoretic, flu shed , nd tre u lou s. (C) Methylp henid te (Rit lin)
(D) Phenytoin (Dil ntin)
25. Which of the follow ing is the ost p p rop ri te (E) Prop r nolol (Ind er l)
ed ic tion to tre t this p tient?
(A) C rb zep ine 29. A 40-ye r-old d ivorced w o n is brought
in to the e ergency d ep rt ent fter being
(B) Clonid ine (C t p res)
fou nd sleep ing in p ile of le ves. Initi lly, she
(C) Lor zep is d ifficu lt to rou se nd h s trou ble nsw er-
(D) Meth d one ing you r qu estions d u e to u bling. She
(E) N ltrexone (ReVi ) d oes not ppe r to be in respir tory d istress,
bu t her p u p ils re noted to be p inp oint. After
26. Withou t the bove tre t ent, w hich of the few hou rs, she beco es ore inter ctive nd
follow ing co p lic tions is he ost t risk for is co p l ining of d iffu se, cr p y bd o in l
d evelop ing? p in nd sy p to s of “the flu .” H er pu p ils
re now slightly d il ted , nd she is y w ning.
(A) Abd o in l p in
H er vit l signs re te p er tu re of 99.3°F,
(B) Cirrhosis he rt r te of 99 be ts/ in, nd blood pressu re
(C) F tty liver of 142/ 90 H g. Cu rrently, w hich of the
(D) Mu scle cr p s follow ing is the ost likely d i gnosis?
(E) Seizu res (A) C nn bis bu se
(B) Coc ine intoxic tion
Questions 27 and 28
(C) Coc ine w ithd r w l
You re w orking in the psychi tric e ergency (D) Op i te intoxic tion
d ep rt ent of l rge etropolit n hospit l. A (E) Op i te w ithd r w l
20-ye r-old n w ith unknow n p sychi tric his-
tory is brou ght in by the p olice fter being fou nd Questions 30 and 31
stu bling n ked rou nd loc l college c p u s.
H e is rked ly git ted , p cing, nd pp e rs to be A 70-ye r-old w id ow is d itted for n ev lu tion
resp ond ing to intern l sti u li. On ex in tion, you of d epression nd nxiety. She tells you th t for
note th t he is t chyc rd ic w ith he rt r te in the the p st 15 ye rs her f ily d octor h s prescribed
110s, he h s tics or sp s s in his f ce, nd he h s “so e p ills” th t h ve help ed her sleep nd feel less
vertic l nyst g u s. nervou s. She s ys th t she r n ou t of the yesterd y
nd since th t ti e h s felt incre singly nxiou s nd
27. Which of the follow ing tests w ou ld be ost jittery. She lso notes th t she’s now h ving tre ors
helpfu l in the d i gnosis? in her h nd s th t h ve not been there before.
(A) CBC
30. Which of the follow ing ed ic tions w ou ld be
(B) Electroencep h logr (EEG) the ost d ngerou s to su d d enly d iscontinue?
(C) M gnetic reson nce i ge (MRI) of the
br in (A) Flu p hen zine (Prolixin)
(D) N oncontr st he d CT (B) I ip r ine (Tofr nil)
(E) Toxicology screen (C) N ortrip tyline (P elor)
(D) Thiorid zine (Mell ril)
(E) Tri zol (H lcion)
172 6: Diffe re ntia l Dia g nos is a nd Ma na g e me nt

nor l. H er hu sb nd confir s th t the p tient h s looks frightened nd sks, “Wh t is h p p ening to


been very w orried bou t “getting d ise se” to the e?” H is ood is “OK” nd his ffect is fl t. H e is
p oint w here she w ill e t only food in se led con- fu lly oriented . H e s ys th t he he rs nu ber of
t iners. H e lso confid es in you th t she h s been p eople t lking to hi , “ ybe in y he d ,” s ying
w orried th t she ight h ve c ncer, nd , d esp ite b d things bou t hi .
re ssu r nces fro her p ri ry c re p hysici n, she
continu es to voice her concerns to her hu sb nd th t 38. Which of the follow ing d i gnoses is ost
“ y intestines re not w orking.” likely?
(A) Brief p sychotic d isord er
36. Which of the follow ing is the ost likely
d i gnosis? (B) Delu sion l d isord er
(C) Schizop hreni
(A) M jor d ep ressive d isord er (MDD)
(D) Schizop hrenifor d isord er
w ithou t p sychotic fe tu res
(E) Su bst nce-ind u ced p sychotic d isord er
(B) MDD w ith p sychotic fe tu res
(C) Obsessive-co p u lsive d isord er (OCD) 39. Which of the follow ing l bor tory tests or pro-
(D) Persistent d ep ressive d isord er ced u res w ould be the ost helpfu l in n rrow -
(d ysthy i d isord er) ing the bove d ifferenti l?
(E) So tic sy p to d isord er
(A) CBC
37. Which of the follow ing ed ic tion(s) w ou ld (B) EEG
be the ost p p rop ri te tre t ent for this (C) Electrolytes
p tient? (D) N oncontr st he d CT
(A) Div lp roex sod iu (E) Toxicology screen
(B) Lithiu nd sertr line
Questions 40 and 41
(C) N ortrip tyline nd lor zep
(D) Sertr line A 35-ye r-old w o n w ith no previous person l or
(E) Sertr line nd risp erid one f ily psychi tric history is brought to the e er-
gency d ep rt ent by her husb nd, w ho reports th t
Questions 38 and 39 his w ife w s tte pting to kill herself by cutting her
w rists. H er husb nd tells you th t 6 onths go the
An 18-ye r-old n is brou ght to the p sychi tric p tient’s gr nd other d ied . Since th t ti e, her hus-
e ergency d ep rt ent by his p rents for ev lu tion b nd believes th t the p tient h s been beco ing
of his beh vior. Three onths go, the p tient st rted ore d epressed . She h s d ifficulty f lling sleep nd
cl sses t the st te university loc ted in d ifferent h s lost 15 lb in 2 onths. She w s recently fired fro
city, lthough he w ou ld co e ho e e ch w eekend her job s p r leg l bec use she w s un ble to con-
to visit. H is p rents h ve noticed th t over the p st centr te nd d e frequent ist kes. She feels guilty
3 w eeks he h s beco e incre singly w ithd r w n nd th t she is un ble to feel better, nd she end orses feel-
d oes not see to be t king c re of hi self. The p r- ings of hopelessness nd w orthlessness. She believes
ents w ere recently c lled by the p tient’s roo te th t her “only w y out of this” is to kill herself.
w ho infor ed the th t the p tient h s not been
going to cl sses for the l st w eek, h s not been e ting 40. Which of the follow ing w ou ld be the ost
or b thing, nd h s been spe king bout how peo- p p rop ri te tre t ent?
p le in his cl sses re trying to kill hi . The p tient
(A) Flu zenil (Ro zicon)
d enies ny d ru g u se nd is not on ny ed ic tion.
On ex in tion, the p tient’s vit l signs re w ithin (B) Flu oxetine
nor l p r eters. H is physic l (inclu d ing neu ro- (C) Flu p hen zine
logic ex in tion) is u nre rk ble. H e is p cing (D) Lor zep (Ativ n)
nd ppe rs to be resp ond ing to intern l sti uli. H e (E) Phenelzine (N rd il)
Que s tions : 36–45 173

41. After being begu n on the p p rop ri te ed ic - the d e n of her college is concerned bout her nd
tion, w hich of the follow ing sid e effects y w orried th t she y be d ep ressed . The p tient tells
i ic w orsening sy pto of her illness? you th t d esp ite being f r w y fro her ho e for
the first ti e, she w s enjoying school nd her new
(A) Ak thisi
friend s u ntil 2 onths go, w hen she le rned th t
(B) Constip tion her p rents w ere getting d ivorced . Since th t ti e,
(C) Di rrhe her gr d es h ve gone fro As nd Bs, to Bs nd Cs
(D) Inso ni bec u se she is concerned bou t her p rents nd her
(E) N u se sister t ho e. She feels “bu ed ou t” ost of the
ti e, nd her friend s note th t she see s unh ppy
Questions 42 and 43 nd occ sion lly beco es te rfu l w hen t lking
bou t her f ily. She d enies d ifficu lty sleep ing or
A 46-ye r-old n w ith no previou s p sychi tric his- ch nges in ppetite or w eight, nd she continu es to
tory p resents w ith co pl ints of feeling d epressed enjoy d ily trip s to the gy to exercise. She d enies
since his w ife d ied 2 w eeks go fro p ncre tic c n- ny suicid l id e tion.
cer. H e sh res th t lthough he know s she is d e d ,
he so eti es he rs her c lling his n e. This h s 44. Which of the follow ing is the ost likely
occu rred sever l ti es nd usu lly h pp ens ore d i gnosis?
often t night w hen he is f lling sleep. H e d enies
ny visu l h llu cin tions, p r noi , or d elu sion l (A) Acu te stress d isord er
beliefs. H e expresses gu ilt bou t not sp end ing ore (B) Ad ju st ent d isord er w ith d ep ressed
ti e w ith his w ife w hen she w s live bu t d enies ood
ny thou ghts of w nting to end his life. H e is now (C) GAD
sleeping only bou t 3 to 4 hou rs t night nd h s (D) MDD
h d slightly d ecre sed pp etite since her d e th.
(E) N or l grief
Desp ite the bove, he h s still been ble to go to
w ork nd focu s on his job.
45. Which of the follow ing tre t ents w ou ld be
ost p propri te?
42. Which of the follow ing is the ost likely
d i gnosis? (A) Di lectic l beh vior l ther p y
(A) Ad ju st ent d isord er (B) Flu oxetine (Proz c)
(B) M jor d ep ressive ep isod e (C) N ortrip tyline
(C) N or l grief (D) Psychother p y
(D) Persistent d ep ressive d isord er (E) Risp erid one (Risp erd l)
(d ysthy ic d isord er)
Questions 46 and 47
(E) Schizop hreni
A 26-ye r-old w o n w ith no p reviou s psychi tric
43. Which of the follow ing w ou ld be the ost history is referred to you by her p ri ry c re p hysi-
pp rop ri te tre t ent for this p tient? ci n for ev lu tion of “ nxiety tt cks.” She tells you
(A) A itrip tyline (El vil) th t p p roxi tely 2 onths go she beg n h ving
period s l sting 10 or 15 inu tes d u ring w hich, she
(B) Flu oxetine (Proz c)
s ys, “I feel like I’ going to d ie.” Du ring these epi-
(C) H lop erid ol (H ld ol) sod es, her he rt r ces, she feels s thou gh she c nnot
(D) Lithiu c tch her bre th, she is d izzy nd fr id she y p ss
(E) N o intervention t this ti e out, nd she h s tingling nd tre ors in her h nd s.
She is concerned bec use she is now h ving d iffi-
Questions 44 and 45 cu lty le ving her hou se d ue to w orry th t these ep i-
sod es w ill occu r, king it i p ossible for her to get
You re sked to see n 18-ye r-old w o n w ith
ho e. She is u n ble to id entify ny triggers le d ing
no p reviou s psychi tric or ed ic l history bec u se
174 6: Diffe re ntia l Dia g nos is a nd Ma nag e me nt

to the episod es. She is not on ny ed ic tion nd (C) Obsessive-co p u lsive p erson lity
h s no ed ic l proble s. d isord er (OCPD)
(D) P nic d isord er
46. Which of the follow ing is her ost likely (E) So tic sy p to d isord er
d i gnosis?
(A) GAD 49. Which of the follow ing tre t ents w ould be
(B) P nic d isord er nd gor p hobi ost ppropri te for this p tient?
(C) Sep r tion nxiety d isord er (A) Cognitive-beh vior l ther p y
(D) Soci l nxiety d isord er (soci l p hobi ) (B) ECT
(E) Sp ecific p hobi (C) Lithiu
(D) Psychod yn ic p sychother p y
47. Which of the follow ing w ou ld be the ost (E) Risp erid one
p propri te tre t ent to initi te?
(A) Alp r zol 50. A 47-ye r-old secret ry co es to your office
(B) Lithiu co pl ining, “I’ lw ys w orried .” She s ys
th t she w orries bout her job, her kid s, her
(C) Prop r nolol
hou sew ork, nd her hu sb nd . She is seeking
(D) Sertr line (Zoloft) help now bec u se she h s been h ving n
(E) Tr nylcyp ro ine (P rn te) incre singly d ifficu lt ti e concentr ting t
w ork nd h s been ore irrit ble w ith p eo-
Questions 48 and 49 ple rou nd her. H er sleep h s been “ok y,”
bu t she d oes not feel rested w hen she gets u p
A 36-ye r-old w o n w ith no for er p sychi t-
in the orning. She h s been ore w re of
ric history is referred to you by d er tologist
these feelings over the l st 2 ye rs nd they
for ev lu tion of her chronic lly ch pped h nd s.
occu r l ost every d y. She d enies ny d is-
She s ys she h s been seeing her d er tologist for
crete p nic tt cks. Which of the follow ing
p proxi tely 5 ye rs for this proble nd tre ted
d i gnoses is the ost likely in this p tient?
w ith v riety of top ic l gents w ith li ited su c-
cess. Over the p st 3 w eeks, her h nd s h ve beco e (A) GAD
w orse, to the p oint w here they re lw ys cr cked (B) OCD
nd bleed ing. Relu ct ntly, she confid es in you th t (C) Soci l nxiety d isord er (soci l p hobi )
she h s h d longst nd ing fe r of ger s, bu t since
(D) P nic d isord er
her colle gu e t w ork h s been sick, she h s been
w shing her h nd s t le st 40 ti es p er d y bec u se (E) Schizop hreni
she is fr id of contr cting the d ise se. She lso
refu ses to tou ch nything th t ight infect her w ith- 51. A 28-ye r-old w o n w ith no p revious psy-
ou t u sing h nd kerchief. She d its to being very chi tric or ed ic l history is d itted to the
tid y t ho e, s w ell. She spend s bou t 2½ hou rs neu rology service for ev lu tion of cu te onset
in the orning getting show ered . She re lizes her of nu bness nd w e kness of the right sid e of
fe rs of cont in tion re irr tion l, but every ti e her f ce nd right r nd leg. Physic l ex -
she tries to stop w shing her h nd s she beco es in tion show s sy etric l 2/ 4 reflexes in
incre singly nxiou s. ll d istribu tions, d ow ngoing p l nt r reflexes
bil ter lly, nd 2/ 5 strength in the right u p per
48. Which of the follow ing d i gnoses w ou ld be nd low er extre ity in ll u scle group s. N o
ost likely? trop hy or f scicul tions re noted . H er g it
is t xic nd st ggering w ith extre e ex g-
(A) GAD ger ted ove ents of her r s; how ever, she
(B) Obsessive-co p u lsive d isord er (OCD) d oes not f ll w hen bu l ting w ithou t ssis-
t nce. Given the severity of her d eficits, she
Que s tions : 46–55 175

see s u nconcerned by her level of d is bility. As.” On ex in tion, she is c l , thin w o n


Which of the follow ing d i gnoses w ou ld be d ressed in very b ggy jogging suit. You co -
the ost p propri te? ent on her thinness, nd she tells you she p rid es
herself on her pp e r nce nd tries to st y sli by
(A) Conversion d isord er
exercising bou t 4 hou rs d y long w ith good
(B) F ctitiou s d isord er d iet. She d enies proble s w ith e ting too u ch or
(C) M lingering too little, nd cl i s th t only d iet nd exercise help
(D) So tic sy pto d isord er her to control her w eight. Desp ite you r concerns,
(E) Und i gnosed neurologic d ise se she d its th t she w ou ld like to lose few ore
p ou nd s. Up on fu rther qu estioning, she infor s you
52. A 54-ye r-old w o n w ith p st ed ic l she h s not h d regu l r enstru l period for over
history of hypothyroid is is d itted w ith ye r.
septic right knee. The surgery te sks you
to ev lu te the p tient bec use they fou nd 53. Which of the follow ing w ou ld be the ost
th t the flu id sp ir te fro the knee w s likely w orking d i gnosis?
grow ing p thogen fou nd p ri rily in the (A) Anorexi nervos
hu n ou th. They su sp ect the p tient w s
(B) Anxiety d isord er
injecting s liv into her knee. You ev lu te the
p tient nd find her to be p le s nt nd coop - (C) Bod y d ys orp hic d isord er
er tive. She tells you th t she h s h d very (D) Bu li i nervos
tough ti e l tely bec use her husb nd h s (E) MDD
recently been sick. Fortu n tely, she is nu rse
nd h s been ble to c re for hi t ho e. 54. Which of the following tests would be ost help-
L tely, she d its to feeling overw hel ed ful in supporting your provision l di gnosis?
nd not pp reci ted . She h s no id e w h t h s
(A) ECG
c used the proble w ith her knee. You t lk to
the f ily nd they tell you the p tient is in (B) Seru yl se
no fin nci l d ifficu lty nd continu es to enjoy (C) Seru gnesiu level
w ork s nu rse. After w orking closely w ith (D) Seru p ot ssiu level
you for sever l w eeks, she eventu lly d its (E) Weight nd height
to injecting her knee lthou gh c nnot u nd er-
st nd w hy she d id it. Which of the follow ing 55. A 5-ye r-old boy is referred to you by his ped i-
is the ost p p rop ri te d i gnosis? trici n for ev lu tion of ggressive beh vior.
(A) Conversion d isord er H is ed ic l nd extensive neu rologic w orku p
w s neg tive. An interview w ith the p tient
(B) F ctitiou s d isord er
reve ls restless boy w ho is ble to eng ge in
(C) Illness nxiety d isord er convers tion. H e tells you he gets ngry nd
(D) M lingering fru str ted in school s he “thinks it’s boring.”
(E) So tic sy p to d isord er H is p rents rep ort th t he is cu rrently rep e t-
ing kind erg rten d u e to p oor p erfor nce
Questions 53 and 54 nd d ifficu lty soci lizing w ith other child ren.
H is ggressive ou tbu rsts t school see to
An 18-ye r-old w o n in her first ye r of college occu r t ti es w hen he d oes not und erst nd
co es to see you for ev lu tion of d epression, fter the schoolw ork. H is other tells you th t she
her roo tes encour ged her to seek help . She still h s to help hi pick ou t his clothes for
rep orts d ifficu lty f lling sleep nd e rly orning school nd get d ressed . H is f ily history is
w kenings, poor concentr tion, f tigue, nd nxi- p ositive for tw o p tern l u ncles w ith le rn-
ety over the p st onth since rriving for the f ll ing d is bilities. Which of the follow ing tests
se ester. She tells you th t her p rents re very strict w ou ld be ost help fu l in the ev lu tion of
nd she is w orried th t she w ill not “get str ight this p tient?
176 6: Diffe re ntia l Dia g nos is a nd Ma na g e me nt

(A) Good enough-H rris Dr w -A-Person p resents w ith od d beh vior, confu sion,
Test blood p ressure of 128/ 85 H g, nd he rt
(B) Kohs Block Test r te of 68 be ts/ in. H is p rents rep ort th t
(C) Minnesot Mu ltip h sic Person lity t ti es they h ve observed hi repetitively
Inventory-2 (MMPI-2) tou ching his sto ch nd r pid ly blinking his
eyes. On ex in tion, he p p e rs d zed nd
(D) Pe bod y Voc bu l ry Test
u n ble to concentr te. Which of the follow ing
(E) Wechsler Intelligence Sc le for Child ren is the ost likely d i gnosis?
(WISC)
(A) A p het ine toxicosis
Questions 56 and 57 (B) Ep ilep sy
(C) H yp oglyce i
An 8-ye r-old boy is brou gh t in to you r office
by h is oth er bec u se sh e is con cern ed th t h e (D) MDD
c nnot stop blinking his eyes nd shru ggin g his (E) So tic sy p to d isord er
sh ou ld ers. Sh e tells you th t this occu rs l ost
every d y n d h s been th is w y for the p st ye r 59. A 50-ye r-old n w ith long history of IV
nd h lf. Sh e tells h er son to stop d oin g this d rug u se is brought to the hosp it l by p olice
w h ile h e is in you r office, n d h e d oes stop . You fter loc l ho eless shelter w orker noted
notice, h ow ever, th t h e st rts b rkin g n d cou gh- hi to be confu sed nd “w lking funny.”
in g rep e ted ly. While t the shelter, he bec e very su spiciou s
of the w orkers nd ccu sed the of t king his
56. Which of the follow ing ed ic tions w ou ld be belongings. On ex in tion, you observe th t
the ost ppropri te in tre ting the p tient’s he h s left p u p il th t cco od tes but d oes
cond ition? not re ct, d epressed d eep tend on reflexes in
ll d istribu tions, nd loss of p osition sense t
(A) Ato oxetine the gre t toes bil ter lly. Althou gh the p tient
(B) Clonid ine w s tre ted w ith ntip sychotic ed ic tions in
(C) L otrigine the p st, he d enies cu rrently t king ny ed i-
(D) Lor zep c tion. Which of the follow ing d i gnoses best
(E) V lp roic cid ccou nts for this p tient’s sy p to s?
(A) Antip sychotic-ind u ced d yskinesi
57. You begin the p p rop ri te ed ic tion nd
(B) Kors koff p sychosis
2 onths l ter the boy nd his other retu rn
(C) N eu rolep tic lign nt synd ro e
for follow -u p visit. She tells you th t w hile
the eye blinking nd should er shrugging h ve (D) N eu rosyp hilis
gotten better, he is now f lling sleep in his (E) Wernicke encep h lop thy
cl sses t school. Wh t is the next ost p pro-
p ri te step in the n ge ent? 60. A 57-ye r-old w o n w ith no p revious p sy-
chi tric history co pl ins of incre sing nxi-
(A) Ad d nother ed ic tion to keep the boy ety over the l st 2 onths. Tod y she reports
w ke. th t it bec e “very b d .” She lso notes th t
(B) Ed u c te the other bou t p rop er sleep w ith these p eriod s of nxiety she gets p ou nd -
hygiene. ing he d che nd once f inted . She continu es
(C) Discontinu e the ed ic tion. to feel “sh ky.” When you check her vit l
(D) Red u ce the d os ge. signs, her he rt r te is 170 be ts/ in nd her
(E) Sw itch to nother ed ic tion. blood p ressu re is 230/ 130 H g. She is d i -
p horetic nd tre u lou s. Given this p tient’s
58. A 17-ye r-old boy w ith h istory of tten - sy p to s, w hich of the follow ing cond itions
tion-d eficit/ h yp er ctivity d isord er (ADH D) is ost likely?
Que s tions : 56–66 177

(A) Acu te lcohol intoxic tion incre singly te rfu l, irrit ble, restless, s w ell
(B) H yperc lce i s h ving d ifficu lty p ying bills nd b l nc-
(C) H yp othyroid is ing her checkbook. She h s occ sion l “hot
fl shes.” On ex in tion, she continuou sly
(D) Pheochro ocyto
ch nges position in her se t. She h s d ifficulty
(E) Posterior circu l tion stroke w ith si p le rith etic nd short-ter e -
ory. H er d eep tend on reflexes re brisk sy -
D IRECTION S (Questions 61 through 66): For each etric lly throu ghou t. She h s fine resting
patient vignette, select the one lettered option tre or of her h nd s nd d ifficu lty rising fro
that is most closely associated w ith it. Each let- se ted p osition.
tered option may be used once, multiple times, or
not at all.
64. A 33-ye r-old w o n p resents w ith 1-ye r
(A) Conversion (fu nction l neu rologic l
history of tension he d ches, recently w ors-
sy pto ) d isord er
ening nd now ssoci ted w ith blu rry vision.
(B) H ep tic enceph lop thy So f r, n extensive neu rologic ev lu tion h s
(C) H u n i u nod eficiency viru s (H IV) been u nreve ling. She volu nteers th t she h s
(D) H yp erthyroid is been sick ost of her life, beginning t ge 16
(E) H yp oglyce i w hen she h d su rgery for p resu ed end o-
(F) Pheochro ocyto etriosis, w hich h s left her u n ble to h ve
sexu l rel tions. Since the su rgery, she h s
(G) So tic sy p to d isord er
h d inter ittent bd o in l cr ping, blo t-
(H ) Syste ic lu p u s erythe tosu s (SLE) ing, nd d i rrhe . Review of her ch rt show s
th t 3 onths go, she s w rheu tologist
61. A 55-ye r-old n is d itted to the hosp it l. for knee, b ck, nd eye p in.
H is w ife reports th t he’s ch nged over the
l st ye r, h s beco e very forgetfu l, nd
h s period s w hen he beco es very u p set. 65. A 30-ye r-old w o n is brou ght to the hos-
On ex in tion, he h s signific nt e ory p it l by her fi ncé. H e s ys th t over the l st
i p ir ent, sterixis, p l r erythe , nd 2 w eeks, she h s been “ tot lly d ifferent p er-
l rge ecchy otic re on his right sc p u l . son.” She h s not been e ting or sleeping, nd
H e w s initi lly cooper tive, bu t now is very h s been irrit ble nd ngry. E rlier in the
git ted nd d e nd s to le ve. onth, she co pl ined of he d ches, p in in
her h nd s nd feet, nd fever. On ex in -
tion, she looks tired ; she is fully oriented bu t
62. A 52-ye r-old n w ith history of IV d ru g h s d ifficu lty rel ting the events of the l st
u se is brou ght to the e ergency d ep rt ent onth. Occ sion lly, she see s to beco e
by his soci l w orker fro ho eless shelter. confused .
She h s know n the p tient for 10 ye rs bu t h s
seen d r stic ch nge in hi over the l st ye r.
While previou sly jovi l nd inter ctive, he is 66. A 42-ye r-old w o n in the e ergency d ep rt-
now d iseng ged nd subd u ed . H e h s been ent pp e rs confu sed nd w ill not llow
incre singly forgetfu l, tod y h ving d ifficu lty blood to be d r w n. She is not oriented to ti e
u sing his e ting u tensils. H e h s lost t le st or pl ce. She t kes ed ic tions for “ cond i-
40 lb in the l st 6 onths nd co pl ined tion,” bu t c nnot el bor te fu rther. The only
bou t feeling w e k nd losing his b l nce. H e vit l signs th t w ere t ken show he rt r te
scores 20/ 30 on the MMSE. of 140 be ts/ in nd blood p ressure of 172/
98 H g. Physic l ex in tion is re rk-
ble for fine tre or of her h nd s bil ter-
63. A 33-ye r-old w o n is brou ght to the e er- lly, nd d i phoresis. Five inu tes l ter, the
gency d ep rt ent by her hu sb nd , w ho tells p tient h s seizu re.
you th t over the l st few onths she h s been
178 6: Diffe re ntia l Dia g nos is a nd Ma na g e me nt

DIRECTIONS (Questions 67 through 72): For each 70. An 86-ye r-old fe le is brou gh t to th e ED
patient vignette, select the one lettered option that is fter the nu rsing ho e st ff noticed su d d en
most closely associated w ith it. Each lettered option ch nge in h er beh vior over the l st sever l
may be used once, multiple times, or not at all. d ys. On history nd p hysic l ex in tion, it
(A) Deliriu is d ifficu lt to elicit infor tion s the p tient
(B) MDD (p seu d od e enti ) keep s f lling sleep . H er vit l signs show
te p er tu re of 101.2°F n d he rt r te of
(C) M jor neu rocognitive d isord er d u e to
101 be ts/ in. Urin lysis is p ositive for
Alzhei er d ise se
ketones.
(D) M jor neu rocognitive d isord er d u e to
frontote por l lob r d egener tion
(Pick d ise se) 71. A 59-ye r-old fe le is brou ght in by her son
w ith w ho she lives. H e st tes th t over the
(E) M jor neu rocognitive d isord er d u e
p st cou p le of onths his other h s been
to H u ntington d ise se
isol tive, no longer sp end s her ti e re d -
(F) M jor neu rocognitive d isord er d ue to
ing w hich she u sed to enjoy. She w ill re in
Lew y bod y d ise se
w ke l te t night nd e rly in the orning,
(G) M jor neu rocognitive d isord er d u e nd her p p etite h s d ecre sed . H e is lso con-
to Prion (Creutzfeld t–J cob) d ise se cerned bec use she w ill so eti es re e ber
(H ) M jor v scu l r neu rocognitive d isord er inor d et ils fro the d y before, bu t other
ti es w ill not rec ll w h t she h d for bre k-
67. A 71-ye r-old fe le is brou ght to the ER by f st. Ment l st tu s ex in tion is signific nt
her hu sb nd . Over the p st few ye rs, he h s for incre sed sp eech l tency, d ep ressed ood
noticed th t she h s been h ving e ory nd ffect, nd trou ble rec lling 2/ 3 ite s
p roble s. She often forgets w here she p u t fter sever l inu tes.
her keys or pu rse. H e no longer lets her d rive
bec u se she gets lost e sily. On MMSE, she h s
72. A 39-ye r-old le w ith no p reviou s p sychi-
p rticul r p roble s w ith the d y of the w eek,
tric history p resents w ith co p l ints of feel-
d te, three-ite rec ll, nd n ing pencil.
ing “very d ep ressed l tely.” H is f ily notes
th t he h s been u ch ore irrit ble recently,
68. A 68-ye r-old le w ith no p sychi tric history often yellin g w ith little p rovoc tion nd
is brou ght by his d u ghter bec u se, d esp ite see ingly t lking to hi self. On MSE, he is
u ltiple re ssu r nces, he continu es to insist noted to h ve qu ick, su d d en, bu t involu nt ry,
so eone else is in the house. The p tient st tes jerking ove ents of his r s.
he sees s ll n every d y ju st insid e of
the d oor. H is d ughter h s lso noticed th t
D IRECTION S (Questions 73 through 98): For each
he h s proble s w ith his e ory. On ex i-
of the multiple-choice questions in this section,
n tion, he h s shu ffling g it nd blu nted
select the lettered answ er that is the one best
ffect, w hich the d ughter st tes beg n t the
response in each case.
s e ti e s the visu l h llu cin tions.
73. A 56-ye r-old n p resents to you r office t
69. A 55-ye r-old le is brou ght by his w ife the requ est of his w ife, w ho s ys th t he d rinks
bec u se she h s noticed ch nges in his beh v- too uch. Wh t w ould be the ost i port nt
ior. L tely, his food p references h ve ch nged str tegy in ev lu ting this p tient for lcohol
nd he is now cu rsing lou d ly in pu blic. On u se d isord er?
ex in tion, he is e sily git ted . N eu rologic
ex in tion d e onstr tes p ositive B binski (A) Ascert in how often he d rinks.
resp onse nd snou t reflex. An MRI show s (B) Ask hi how frequ ently he gets d ru nk.
trophy in the front l nd te p or l lobes. (C) Ask hi w h t his f ily nd friend s
s y bou t his d rinking.
Que s tions : 67–78 179

(D) Perfor co plete l bor tory Questions 77 and 78


investig tion.
A 55-ye r-old le w ith history of lcohol u se
(E) Qu ntify the ver ge ou nt he d rinks.
d isord er presents to you r office bec use he w ould
like to stop d rinking lcohol. H e believes he d rinks
74. A 42-ye r-old d ivorced le p resents to you r
bec u se he is d ep ressed nd w ishes to lso get tre t-
office st ting th t for the p st 7 onths he
ent for his d epression. After fu rther history is
h s been w orrying bout fin nci l nd w ork-
obt ined , it is reco end ed th t the first step is to
rel ted p roble s. H e st tes th t ore often
bst in fro ll lcohol s it c n c u se d ep ression.
th n not, he is w orrying bou t these issues
H e grees to this pl n.
throu ghou t the d y. H e h s d ifficu lt ti e
st ying sleep, he feels tired ll d y long,
77. Wh t is the likelihood of his re ining
nd he c n no longer concentr te d equ tely
d ep ressed if he is ble to refr in fro u sing
t w ork d u e to the w orrying. H e lso tells
lcohol for 1 onth?
you th t he feels very tense in his shou ld er
nd b ck. Which of the follow ing is the ost (A) 5%
p propri te d i gnosis? (B) 15%
(A) Depend ent person lity d isord er (C) 25%
(B) Gener lized nxiety d isord er (GAD) (D) 33%
(C) M jor d ep ressive d isord er (MDD) (E) 50%
(D) P nic d isord er
78. The bove p tient retu rns fter 6 w eeks of
(E) Soci l nxiety d isord er
int ining sobriety fro lcohol. H e tells
you th t he h s continu ed to feel d ep ressed .
Questions 75 and 76
H e h s p roble s sleep ing t night, feels tired
You re interview ing 54-ye r-old rried w o n throughou t the d y, h s poor pp etite, nd he
w ho h s been u rged to “see shrink” by her f - no longer d erives ple su re fro pl ying w ith
ily. She d escribes sy p to s of feeling ineffectu l, his d ogs? Which of the follow ing ed ic tions
believing th t the w orld is lw ys hostile to her, nd w ou ld be the ost p p rop ri te to begin t this
know ing th t things w ill never ch nge. ti e?
(A) A itrip tyline (El vil)
75. This tri d of sy p to s is ost ssoci ted
(B) Bu sp irone (Bu Sp r)
w ith w hich of the follow ing d isord ers?
(C) Lithiu (Esk lith)
(A) Dep ressive d isord ers (D) Lor zep (Ativ n)
(B) Dissoci tive d isord er (E) Sertr line (Zoloft)
(C) GAD
(D) P nic d isord er Questions 79 and 80
(E) Schizop hreni
A 52-ye r-old le w ith longst nd ing history of
lcohol use d isord er is brought into the ED intoxi-
76. Which of the follow ing tre t ents w ou ld
c ted w ith lcohol. B sic l bs re d r w n nd he is
ost likely t rget these sy p to s?
found to be d ehyd r ted , so he is given IV fluid s
(A) Beh vior l ther p y long w ith glu cose. Abou t n hou r l ter, the physi-
(B) Cognitive ther p y ci n reev lu tes the p tient nd find s th t his speech
(C) Cou p les ther p y is ore slurred , he is very confused , nd he now h s
nyst g us.
(D) Interp erson l ther p y
(E) P r d oxic l ther p y
180 6: Diffe re ntia l Diag nos is a nd Ma na g e me nt

79. Which of the follow ing is the next ost (A) Ad ju st ent d isord er
p propri te step in the n ge ent of this (B) Bip ol r d isord er
p tient? (C) MDD
(A) Ad inister ntibiotics. (D) Persistent d ep ressive d isord er
(B) Ad inister benzod i zep ine. (E) Schizop hreni
(C) Ad inister hep rin.
(D) Ad inister thi ine. 83. A 45-ye r-old n is in the e ergency dep rt-
ent bec use of di betic foot ulcer. In g ther-
(E) Ad inister tissu e p l s inogen
ing history, the physici n le rns th t this n
ctiv tor.
lives lone nd works nights s security gu rd.
He s ys he h s no friends but th t this does not
80. After d in istr tion of th e correct ed ic -
bother hi . He h s never been hospit lized
tion the p tien t i p roves signific ntly. H is
or received ny psychi tric help. On MSE, his
f ily rrives th e n ext orn in g n d th ey
ffect is fl t. Although he nswers questions
tell you th t he h s recen tly been st rting
nd see s to trust the judg ent of the doctors,
to ke u p f cts bou t h is life. You ord er
he h s little interest in the interview. He exhibits
n MRI of his br in nd d iscover lesion s in
no signs or sy pto s of psychosis or depres-
h is ill ry bod ies. Wh ich of th e fol-
sion. Wh t is the ost likely di gnosis?
low in g d i gn oses w ou ld best exp l in th ese
sy p to s? (A) MDD
(A) Alcohol-ind u ced jor neu rocognitive (B) P r noid p erson lity d isord er
d isord er (Kors koff synd ro e) (C) Schizoid p erson lity d isord er
(B) Conversion d isord er (D) Schizop hreni
(C) F ctitiou s d isord er (E) Schizotyp l p erson lity d isord er
(D) M jor neu rocognitive d isord er d u e to
Alzhei er d ise se 84. The id entified p tient is 30-ye r-old sep -
r ted fe le brou ght into the e ergency roo
(E) Wernicke encep h lop thy
by her id entic l tw in sister. The p tient’s his-
tory is not ble for prior episod e of d epres-
Questions 81 and 82
sion 5 ye rs go su ccessfu lly tre ted w ith
A 27-ye r-old n co p l ins th t he h s felt “d ow n venl f xine. The p tient h s been st ying
in the d u ps” for onths nd is feeling gu ilty w ith her sister since her sep r tion 1 onth
bec u se he h s been h ving n extr rit l ff ir. go. For the p st 2 w eeks, she h s been p cing
In recent w eeks, he h s st rted to believe th t his rou nd the hou se, not sleep ing ore th n 2 to
w ife is p oisoning his food nd the rest of his f ily 3 hou rs per night. Desp ite her feeling “s d ”
is involved in n el bor te plot to d rive hi fro i ed i tely fter the sep r tion, the p tient
the hou se. now feels “w ond erfu l, like I c n cco p lish
nything!” In f ct, she h s been tte p ting to
81. Assu ing his thinking is d elu sion l, how re od el her sister ’s b throo , even thou gh
w ou ld his d elu sions be best ch r cterized ? she h s no tr ining or experience. H er sister
h s been extre ely concerned bou t her, bu t
(A) Biz rre she h s been u n ble to t lk to her bout it s,
(B) Ego-syntonic “I c n’t get w ord in ed gew ise, nd she
(C) Mood congru ent d oesn’t lw ys ke sense.” The p tient is
(D) Mood incongru ent only t king or l contr ceptives nd o epr -
(E) So tic zole for cid reflux. H er sister is concerned
th t she, herself, y eventu lly d evelop this
82. Which of the follow ing d i gnoses w ou ld ost illness. Wh t is her p p roxi te risk of d evel-
likely be resp onsible for the bove d elu sions? oping this d ise se?
Que s tions : 79–89 181

(A) 0% to 10% ed ic tions w ou ld be the best choice for this


(B) 10% to 20% p tient?
(C) 20% to 30% (A) Alp r zol (X n x)
(D) 50% to 70% (B) Cod eine (Brontex)
(E) 80% to 90% (C) Dip henhyd r ine (Ben d ryl)
(D) Qu eti p ine (Seroqu el)
85. A 33-ye r-old w o n w ith history of
(E) Zolp id e (A bien)
schizop h reni tells you she h s been h e ring
voices for the p st 5 ye rs. Sh e h s b se-
88. An 8-ye r-old girl p resents to her p ed i tri-
line u d itory h llu cin tion th t she c nnot
ci n w ith her p rents second ry to incre sed
u nd erst n d ost of the ti e. Du ring p eri-
p roble s t school. She recently entered the
od s w here the voices beco e w orse, she
third gr d e nd h s been h ving d ifficu lties
typ ic lly he rs co nd u d itory h llu ci-
w ith beh vior in cl ss. At ho e, her p r-
n tions. Fro s fety st n d p oint, w hich of
ents d escribe her s ctive nd lw ys ov-
the follow ing is the ost i p ort nt infor-
ing fro one ctivity to the next. At school,
tion to obt in reg rd ing her u d itory
the child is u n ble to sit still in her ch ir nd
h llu cin tions?
frequ ently interru p ts the lessons. On ex i-
(A) H ow long she h s been he ring voices. n tion, you notice th t she is fid gety nd
(B) H ow lou d the voices re. const ntly p l ying w ith her h nd s. When
(C) Wh t the voices re s ying. she tte p ts to co p lete t sk she is e sily
d istr cted . Which of the follow ing w ou ld be
(D) Whether she recognizes the voices.
the ost effective p h r cother p y for this
(E) Whether the voices co e fro insid e or p tient?
ou tsid e her he d .
(A) Clonid ine
86. A 38-ye r-old n co p l ins th t for the (B) I ip r ine
p st 2 ye rs he h s requ ired sever l n p s (C) Methylp henid te
over the cou rse of the d y; he find s the n ps (D) Risp erid one
qu ite refreshing, but sees his d octor bec use
(E) Sertr line
l tely, s he is w king u p , he feels o en-
t rily “p r lyzed .” H e is cu rrently not on ny
89. A 30-ye r-old le co es to n urgent c re
ed ic tions. H e d enies the u se of ny illicit
f cility 8 hou rs fter the onset of u scle ches,
su bst nces. Which of the follow ing is the ost
ru nny nose, runny eyes, d ifficu lty sleeping,
likely d i gnosis?
nd loose stools th t he ttribu tes to flu
(A) Circ d i n rhyth sleep –w ke d isord er, viru s. H e lives in d ifferent city nd is visit-
d v nced sleep -ph se type ing for conference. H e rep orts th t he forgot
(B) Circ d i n rhyth sleep –w ke d isord er, his ed ic tions t ho e. H e t kes orphine
d el yed sleep -p h se typ e IR every 6 hou rs for chronic low b ck p in,
(C) H yp erso nolence d isord er d ip henhyd r ine occ sion lly to help w ith
sleep , venl f xine 150 g d ily for d ep res-
(D) N rcolepsy
sion nd Lisinopril for blood pressu re control.
(E) P r so ni On ex in tion, he is sw e ting, y w ning, h s
d il ted pu pils nd is ss ging his u scles.
87. A 45-ye r-old le w ith history of op i te H is blood p ressu re is 160/ 95 H g, pu lse
u se d isord er p resents to you r office co p l in- of 102 bp , nd he is febrile. When sked to
ing of p roble s w ith sleep . H e cl i s th t cont ct his pri ry c re p hysici n he beco es
he h s been sober fro heroin for the p st incre singly git ted , st ting they w ou ld not
10 ye rs. H e sks to be st rted on ed ic - contribute ny infor tion to this visit. L b
tion for his inso ni . Which of the follow ing v lues re ll w ith in nor l li its. Which
182 6: Diffe re ntia l Diag nos is a nd Ma na g e me nt

of the follow ing ost likely exp l ins his etfor in, u ltivit in, n OTC herb l su pp le-
sy pto tology? ent, tr d ol for chronic b ck p in, nd recent
ntibiotic prescription for linezolid . H is vit l signs
(A) Anticholinergic toxicity
re: BP of 160/ 92, he rt r te of 110 bp , nd te -
(B) H ypertensive crisis p er tu re of 101.2°F. On ex in tion, he d i p horetic
(C) Influ enz nd h s clonu s in his bil ter l low er extre ities.
(D) Op i te w ithd r w l
(E) Serotonin synd ro e 92. Which of the follow ing is his ost likely
d i gnosis?
Questions 90 and 91
(A) Bipol r d isord er
A 28-ye r-old fe le w ith p st ed ic l history (B) H ypertensive crisis
of hypothyroid is nd syste ic lu pu s erythe - (C) Migr ine w ith u r
tosus (SLE) is hospit lized for lupu s fl re requ ir- (D) N eu rolep tic lign nt synd ro e (N MS)
ing IV steroid s. You re consu lted for the p tient
(E) Serotonin synd ro e
s she is co pl ining of ud itory h llu cin tions
nd h s been beco ing incre singly git ted . H er
93. Which of the follow ing is the best next step in
ed ic tions inclu d e IV ethylp red nisone, u lti-
the n ge ent of this p tient?
vit in, nd levothyroxine. On ex in tion, the
p tient’s ood is l bile nd she end orse u d itory (A) Continu e ll ho e ed ic tions.
h llucin tions. H er vit l signs re st ble. H er TSH is (B) Discontinu e linezolid , tr d ol,
0.7 IU/ L nd her T4 is 7.2 u g/ d L. p henelzine.
(C) Discontinu e etfor in.
90. Which of the follow ing is the ost likely
(D) Discontinu e over the cou nter herb l
d i gnosis?
sup ple ent.
(A) Bipol r d isord er d u e to hyperthyroid is (E) Incre se risp erid one.
(B) Brief psychotic d isord er
(C) MDD w ith p sychotic fe tu res 94. A 58-ye r-old le w ith no p st ed ic l his-
(D) Schizop hreni tory is brou ght to the d octor ’s office by friend .
(E) Steroid -ind u ced p sychotic d isord er The friend rep orts the p tient h s ccu u -
l ted too u ch “stu ff” in his one-story ho e
91. Which of the follow ing is the best next step in nd h s not been ble to le ve his ho e. The
the n ge ent of this p tient? p tient cl i s th t he y need these ite s t
so e point in the futu re nd refu ses to throw
(A) Decre se steroid d os ge nything w y. When sked if he h s ever
(B) Incre se levothyroxine tried to cle n out his hou se he gets d efensive,
(C) Initi te ECT fid gety nd st tes he cou ld “ bsolutely never
(D) Initi te lithiu throw nything ou t!” H is friend brings in p ic-
tu res of the p tient’s ho e show ing ound s
(E) Initi te sertr line
of ite s in every roo . The p tient h s been
u ne p loyed for 1 ye r, expl ining th t he
Questions 92 and 93
need s the ti e to t ke c re of his ho e. Which
A 38-ye r-old n w ith p st ed ic l history of of the follow ing is the ost likely d i gnosis?
chronic low er b ck p in nd hypertension presents (A) GAD
w ith friend to the e ergency roo co p l ining of
(B) H o rd ing d isord er
git tion nd he d che for 2 d ys. Upon fu rther
qu estioning, he d its to p st p sychi tric history (C) OCD
of MDD w ith psychotic fe tures. Soon fter p resen- (D) Obsessive-co pu lsive person lity
t tion he beco es confu sed nd nonresponsive. H is d isord er (OCPD)
ed ic tion list inclu d es p henelzine, risp erid one, (E) P nic d isord er
Que s tions : 90–100 183

95. A 31-ye r-old fe le is fou nd in D ll s, Tex s 98. A 35-ye r-old le is u rged to go to psy-
sitting in coffee shop for n entire d y. When chi trist by cow orker bec u se of frequent
sked to le ve the coffee shop u p on closing, nger ou tbu rsts. H e rep orts th t s long s
the p tient c nnot rec ll ny id entifying infor- he c n re e ber he h s h d “hot te p ter.”
tion or w here she is fro . The p olice re L tely, he h s been getting into to trou ble t
c lled , w ho escort her to n e ergency d ep rt- w ork d u e to u ltip le verb l confront tions
ent. When se rched , her ID c rd is fou nd w ith cow orkers, w hich occ sion lly esc l te
in her p u rse, d ocu enting th t the p tient is to the p tient throw ing v rious office objects
fro Michig n. Fu rther investig tion reve ls cross the roo . H e feels very gu ilty fter, but
her f ily rep orted her issing 2 w eeks go. he is u n ble to p revent his beh vior. H e h s no
Of note the p tient w s d op ted t you ng signific nt leg l proble s otherw ise. Which of
ge second ry to child bu se. Wh ich of the the follow ing is the ost likely d i gnosis?
follow ing is the ost likely d i gnosis?
(A) Antisoci l p erson lity d isord er
(A) Bord erline p erson lity d isord er (B) Bip ol r II d isord er
(B) Dep erson liz tion d isord er (C) Bord erline p erson lity d isord er
(C) Dissoci tive nesi (D) Cond u ct d isord er
(D) Dissoci tive id entity d isord er (E) Inter ittent exp losive d isord er
(E) M jor neu rocognitive d isord er d u e to
Alzhei er d ise se D IRECTION S (Questions 99 through 108): For
each of the follow ing patients, choose the most
Questions 96 and 97 appropriate diagnosis. Each lettered option may
be used once, multiple times, or not at all.
A 26-ye r-old fe le p resents to her pri ry c re
physici n w ith d ifficu lty sleep ing. She co pl ins (A) Antisoci l p erson lity d isord er
of const ntly fid geting in bed w ith const nt urge
(B) Avoid nt p erson lity d isord er
to ove her legs. She gets u p nd w lks rou nd in
ord er to help relieve it. This occu rs ost nights of (C) Bord erline p erson lity d isord er
the w eek nd h s gre tly interfered w ith her bil- (D) Dep end ent p erson lity d isord er
ity to sleep . Physic l ex in tion is neg tive for ny (E) H istrionic p erson lity d isord er
bnor lities. H er l b w ork show s low MCV, low (F) N rcissistic p erson lity d isord er
ferritin, nd he oglobin of 9.0 g/ d L. (G) Obsessive-co p u lsive p erson lity
d isord er
96. Which of the follow ing is the ost likely
(H ) P r noid p erson lity d isord er
d i gnosis for this p tient?
(I) Schizoid person lity d isord er
(A) GAD (J) Schizotyp l person lity d isord er
(B) Inso ni d isord er
(C) N on-REM sleep rou s l d isord er 99. A 45-ye r-old w o n recently is rele sed fro
(D) Obstru ctive sleep p ne p rison for r ed robbery. She h s nu erous
(E) Restless legs synd ro e j il sentences for ste ling, bribery, nd rson.
She end orses th t she s w p sychi trist s
97. Which of the follow ing is the ost p p rop ri- child for “beh vior l proble s,” getting into
te step in the tre t ent of this p tient? trou ble t school nd t ho e. Upon further
questioning, she rec lls p eriod of ti e
(A) Initi te hyd rocod one. torturing her f ily’s p et c t s child .
(B) Initi te iron rep l ce ent.
(C) Initi te lor zep . 100. A 57-ye r-old n presents to his pri ry
(D) Initi te el tonin. p hysici n w ith co p l ints of bd o in l p in.
(E) Initi te risp erid one. H e is cco p nied by his other, w ith w ho
184 6: Diffe re ntia l Diag nos is a nd Ma na g e me nt

he lives. The p tient ppe rs to d efer to his see the Chief of the d ep rt ent, nd how he
other to nsw er ny of the qu estions d u r- c nnot be “bothered ” by low -level e ployees.
ing the interview. The p tient t lks bou t how
he c lls his other throughou t the w orkd y 105. A 46-ye r-old n is reco end ed to see
to sk her trivi l qu estions nd so she c n help e p loyee he lth s he continu ou sly believes
hi ke ny, everyd y d ecisions. H e h s his colle gu es t w ork re ou t to s bot ge
recently been ore nxiou s s he fe rs th t she his e p loy ent. As resu lt, he h s w ritten
is “old er” nd y d ie soon, “w hich w ou ld thre tening e ils to his peers nd su pervi-
le ve e lone, help less, nd u n ble to cop e.” sors. When confronted w ith the f ct th t he
h s been invited to t ke p rt in soci l ctivi-
101. A 30-ye r-old n w orks in lighthou se d u r- ties w ith others, he st tes th t he is being
ing the overnight shift by hi self. H e h s “tricked into co p l cency” nd pl ns to sue
no friend s, prefers to be lone in his sp re the org niz tion.
ti e, nd d oes not d esire to ke person l
or inti te rel tionships. H e presents d ue to 106. A 21-ye r-old w o n w ho is in her third ye r
incre sed stress now th t he h s cow orker t of college p resents to ther p ist u p set th t
his job. she is u n ble to ke friend s. She exp resses
th t she w ou ld like to join school clu b to
102. A 34-ye r-old w o n h s just gone throu gh eet new p eop le nd ke friend s bu t she is
her second d ivorce nd is cu rrently in too shy. She d oesn’t go ou t to p rties d esp ite
tu u ltu ous rel tionship w ith n she et being sked , bu t “they w ou ld n’t like e
1 onth go. H er ex in tion reve ls sc rs nyw y.”
on her w rist, w hich she expl ins re fro cut-
ting herself “w hen I get ngry or up set.” She 107. A 34-ye r-old w o n is new ly d i gnosed w ith
often feels e pty insid e bu t d its to intense hyp ertension. Upon interview, she d its
ch nges of ood , qu ickly beco ing d espon- th t she st ys l te t her job on regul r b sis
d ent or ngry. She reports it is ll her f u lt she bec use she spend s n excessive ount of
feels this w y. ti e on e ch project, ensuring everything is
in ex ct ord er nd properly for tted . She
103. A 45-ye r-old w o n presents to her p ri ry triple-checks not only her ow n but ll of her
c re ppoint ent p rovoc tively d ressed for colle gues’ w ork. She d oesn’t believe th t oth-
rou tine ex in tion. She exu ber ntly exp l ins ers re “c rrying their lo d ” but believes she is
her cu rrent rel tionship w ith so eone she h s the only one w ho excels t her job. She is d is-
et only online nd not in person. She goes ppointed , how ever, bec use her fellow col-
into gre t d et il reg rd ing his erits, cl i ing le gues re ll ble to le ve t the ppropri te
th t “he is the one.” H ow ever, t the end of the ti e.
interview she sks the p hysici n if he is single.
108. A 32-ye r-old w o n p resents to her OB/
104. A 32-ye r-old n p resents to the e ergency GYN for routine c re. She is noted to be
d ep rt ent fter suffering l cer tion to his d ressed w ith ulticolored sc rves, b ngles,
left ind ex finger. When ed ic l stu d ent nd l rge nu ber of cryst l jew elry. She
tte p ts to ev lu te hi he rebu kes hi , st tes she is p l re d er s profession,
excl i ing “I only t lk to ttend ing p hysi- believing she h s the tools to u nlock p er-
ci ns.” H e is noted to be t lking lou d ly on his son’s fu tu re. She c e to her p point ent
telephone bout how su ccessful he h s been fter re d ing her strologic l forec st, bu t w s
in the fin nci l rket, how he is su re he w ill concerned bec u se of the “color of y ur .”
CHAPTER 7

Prac tic e Te s t 1
Que s tions

D IRECTION S (Questions 1 through 107): For each Questions 3 and 4


of the multiple-choice questions in this section,
select the lettered answ er that is the one best A 44-year-old w om an p resents to her p rim ary care
response in each case. d octor w ith m u ltip le com p laints, inclu d ing w eak-
ness in her low er extrem ities, bloating, head aches,
interm ittent loss of ap p etite, and back p ain. A care-
Questions 1 and 2 fu l review of sym p tom s reveals m any other vagu e
sym p tom s. H er com p laints d ate back to ad oles-
A 29-year-old m an w ith a history of bip olar d isor-
cence and she has seen m any d octors. Thorou gh
d er presents to the psychiatric em ergency d ep art-
w orku p s, inclu d ing an exp loratory lap arotom y,
m ent saying that he is the king of “Pum bar” and
have failed to u ncover any clear, anatom ic, or
need s everyone’s allegiance for the u p com ing w ar
p hysiologic cau se.
w ith the Martians. In the p ast few d ays, he has slep t
a total of 3 hou rs bu t says that he is not tired . H e has
3. Which of the follow ing is the best ap proach to
spent all of his m oney soliciting phone sex. N ow , he
this patient?
is agitated , d em and ing, and threatening.
(A) Tell her any physical w orku p is
1. Which of the follow ing is the best treatm ent u nnecessary.
for this p atient in the acu te setting? (B) Tell her to com e back in 1 m onth and , if
(A) Carbam azep ine (Tegretol) the sym ptom s are still present, you w ill
initiate a p hysical w orku p .
(B) Divalp roex sod iu m (Dep akote)
(C) Tactfu lly ask her w hy she is inventing
(C) H alop erid ol (H ald ol)
sym ptom s.
(D) H yd roxyzine (Atarax)
(D) Assess her for other p sychiatric
(E) Lithiu m illnesses.
(E) Initiate a p hysical w orku p and arrange
2. After treating the p atient acu tely, a m ed ica-
for follow -u p in a year’s tim e.
tion is requ ired for ongoing treatm ent of his
bip olar d isord er. You find that he has a history
4. Which of the follow ing p ersonality d isord ers
of agranulocytosis. Which of the follow ing
w ou ld m ost likely be com orbid in the above
m ed ications w ou ld be the best choice for his
p atient?
treatm ent?
(A) Avoid ant
(A) Antip sychotic m ed ication
(B) Bord erline
(B) Carbam azep ine
(C) Obsessive-com p u lsive
(C) Divalp roex sod iu m
(D) Schizoid
(D) Lithiu m
(E) Schizotyp al
(E) Lorazep am (Ativan)

205
206 7: Pra c tic e Te s t 1

Questions 5 and 6 7. Which of the follow ing sym p tom s is consi-


d ered a “negative sym ptom ” in this patient’s
A 50-year-old sin gle m an h as in creasin gly becom e p resentation?
a con cern to his n eigh bors. H e w orks at a com ic-
book store, d resses in od d , ou td ated cloth es, an d (A) Au d itory hallu cinations
d isp lays p oor eye con tact. Althou gh h e tend s (B) Delu sions
to keep to him self, h e h as told neighborhood chil- (C) Flat affect
d ren th at th ere are w itch es w h o live d ow n th e (D) Loose associations
street.
(E) Paranoia
5. Which of the follow ing is the m ost likely 8. The p atient is started on m ed ication and
d iagnosis? m any of her sym p tom s im p rove. She begins
(A) Bipolar d isord er a new job and d oes w ell. One year later, she
(B) Bord erline p ersonality d isord er is brou ght to her d octor florid ly p sychotic,
actively hearing voices, and extrem ely p ara-
(C) Schizoid p ersonality d isord er
noid . She believes that her boss is trying to kill
(D) Schizop hrenia her. She has an u p p er respiratory viral illness
(E) Schizotyp al p ersonality d isord er that she believes to be the w ork of a foreign
governm ent. She d iscontinu ed her m ed ica-
6. The patient’s brother brings him to a d octor tion 4 w eeks ago becau se she felt too sed ated .
because, since the d eath of their mother, the In the p ast year, her cigarette sm oking habit
patient’s paranoia has cau sed him to qu estion has d ecreased to one p ack p er d ay. What is the
his neighbors’ activities. H e has since m oved m ost likely cau se of her exacerbation?
into a hotel that he could not afford in ord er
to get aw ay from the “spies” living next d oor. (A) A reaction to the viral illness
Which of the follow ing is the most appropriate (B) Decreased cigarette sm oking
intervention? (C) Med ication noncom p liance
(A) Antip sychotic (D) Stress from w ork
(B) Benzod iazep ine (E) Treatm ent refractory illness
(C) N o treatm ent
Questions 9 and 10
(D) Psychod ynam ic p sychotherapy
(E) Selective serotonin reu p take inhibitor A 44-year-old man complains to his doctor that he is
(SSRI) alw ays tired and is having d ifficulty getting out of
bed in the morning. Upon questioning, he reveals he
Questions 7 and 8 has three or four d rinks each night and perhaps more
on the w eekend s, but d enies he has any problem w ith
A 25-year-old fem ale college grad u ate is brou ght alcohol. A d iagnosis of alcohol use d isord er is mad e
to her d octor by her m other. Described as “od d ” and the patient comes to your office in acute alcohol
since she lost her job a year ago, the p atient has w ithd raw al. He subsequently has a w ithd raw al sei-
com p lained of hearing voices and believes that zure and is ad mitted to the intensive care unit.
her bod y is receiving Wi-Fi com m u nications for a
cou nterterrorist op eration. H er m other notes she 9. Which of the follow ing laboratory find ings
has been isolating herself in her room . She is alert w ou ld be m ost likely fou nd in this p atient?
and oriented bu t su sp iciou s and gu ard ed on exam -
ination. H er affect is flat and her sp eech reveals (A) Decreased p rothrom bin tim e
loose associations. A com p lete m ed ical w orku p is (B) Elevated or d ep ressed liver enzym es
negative. (C) H igh blood -alcohol level
(D) H yp erm agnesem ia
(E) Throm bocytosis
Que s tions : 5–14 207

10. Which of the follow ing m ed ications w ou ld be to w orsening d epressive sym ptom s and acute sui-
m ost im portant to ad m inister? cid ality. She has had m u ltiple trials of m ed ications
w ithou t significant im p rovem ent. A cou rse of elec-
(A) Diazepam
troconvulsive therap y (ECT) is d eterm ined to be the
(B) H alop erid ol next ap p ropriate step.
(C) Lorazep am
(D) Phenytoin 12. While u nd ergoing the ECT, the treatm ent
(E) Valp roic acid team w ishes to m onitor im provem ent in her
d epression. Which of the follow ing tests has
11. A 28-year-old woman is brought to the emer- the greatest reliability and valid ity for this
gency department for active suicidal ideation p u rp ose?
with a plan to overdose on acetaminophen. (A) Beck Dep ression Inventory
She has no history of a psychiatric illness but
(B) Draw -A-Person Test
endorses many criteria for major depressive
disorder (MDD), including poor sleep for the (C) H alstead -Reitan N eu rop sychological
past 2 weeks. She recently lost her job and Battery
fears that she may not be able to pay her rent. (D) Rorschach Test
Attempts to obtain collateral information have (E) Them atic Ap p ercep tion Test (TAT)
been unsuccessful. You believe the patient
requires inpatient evaluation but her insurance 13. After one of her treatm ents, the p atient com -
company denies authorization for inpatient p laints of m em ory im p airm ent. Which of the
care, alternatively authorizing eight outpatient follow ing tests w ou ld be the m ost ap p rop riate
visits with a psychiatrist. You speak to the to assess her com plaint?
weekend on-call physician-reviewer and report
(A) Beck Dep ression Inventory
that the patient remains unsafe and wishes to
be discharged. Upon learning that the patient (B) Brow n-Peterson Task
does not have a history of psychiatric illness, the (C) Bu lim ia Test—Revised
reviewer fails to authorize inpatient care, despite (D) Eating Disord er Inventory 2 (EDI-2)
your assessment. Which of the following is the (E) State-Trait Anxiety Inventory
most appropriate intervention?
(A) Ad m inister an antipsychotic m ed ication 14. A 67-year-old w oman w ith a history of
and reevalu ate the p atient in 1 hou r. d epression presents to your office for evaluation.
Her symptoms of poor appetite, insomnia,
(B) Ad m it the p atient on an em ergency
low energy, and feelings of hopelessness have
certificate to an inpatient facility.
w orsened recently. She has been on several
(C) Begin antid ep ressant therap y and d ifferent serotonin-specific reuptake inhibitors
arrange for outp atient follow -u p the (SSRIs), w hich you learn have not resulted in
next d ay. complete remission of her symptoms. Which of
(D) Explain to the p atient that her the follow ing med ications w ould be the most
insu rance com p any d id not au thorize appropriate to prescribe?
hosp italization and d ischarge her w ith
follow -u p care. (A) Citalopram
(E) Prescribe a m ed ication to help her (B) Flu voxam ine
sleep, arrange for follow -up care, and (C) Paroxetine
d ischarge her from the hospital. (D) Sertraline
(E) Venlafaxine
Questions 12 and 13

A 34-year-old w om an suffering from severe m ajor


d epressive d isord er is ad m itted to the hospital d u e
208 7: Pra c tic e Te s t 1

Questions 15 and 16 (D) H yp ochond riasis


(E) Panic d isord er
A 29-year-old w om an p resents to the em ergency
d ep artm ent w ith her 3-year-old child reporting that
18. Which of the follow ing m ed ications is the m ost
the child suffered a seizu re w hile at hom e. H ospi-
efficaciou s in the treatm ent of this illness?
tal record s verify that this is the third em ergency
d ep artm ent visit in as m any w eeks for the sam e (A) Clonid ine (Catap res)
p resentation. N eu rologic w orku p for seizu re d is- (B) H alop erid ol
ord er w as negative. Initiation of an anticonvu lsant (C) Lithiu m
has been ineffective. The m other becom es very fru s-
(D) Prop ranolol
trated , d em and ing that her son be ad m itted to the
hosp ital for further testing. (E) Sertraline (Zoloft)

15. Based on the above p resentation, w hich of 19. A w om an being treated for m ajor d ep ression
the follow ing d iagnoses is m ost likely in the is brou ght to the em ergency d ep artm ent after
child ? being fou nd u nconsciou s by a neighbor. The
neighbor states that over the p ast few d ays
(A) Conversion d isord er the w om an had been com plaining of severe
(B) Factitiou s d isord er head aches. She also m entions that the w om an
(C) N o d iagnosis often enjoys red w ine. The w om an’s blood
(D) Seizu re d isord er pressu re is record ed as 220/ 110 m m H g.
Which of the follow ing shou ld be ad m inistered
(E) Sep aration anxiety d isord er
im m ed iately?
16. The m other is ad d itionally m ost likely to su f- (A) Alp ha-blocker
fer from w hich of the follow ing? (B) Beta-blocker
(A) Bipolar d isord er (C) Brom ocrip tine (Parlod el)
(B) Ep ilep sy (D) Calciu m channel blocker
(C) Major d ep ressive d isord er (E) Dantrolene sod iu m (Dantriu m )
(D) Posttrau m atic stress d isord er (PTSD)
Questions 20 and 21
(E) Schizop hrenia
A 38-year-old m an w ith schizop hrenia has had
Questions 17 and 18 nu m erou s hosp italizations and m any trials w ith
variou s antip sychotic m ed ications. H e continues to
A 26-year-old m an is being evalu ated in the em er-
be sym p tom atic, w ith d erogatory au d itory hallu ci-
gency d ep artm ent for su d d en onset of chest p res-
nations, p aranoia regard ing his neighbors, and poor
sure and d yspnea. This is his third em ergency
self-care. H e d enies su icid al or hom icid al id eation.
d ep artm ent visit for sim ilar sym p tom s for w hich he
H e has recently been started on clozap ine (Clozaril),
rep orts “I feel like I’m going to d ie.” An electrocar-
w ith significant im p rovem ent in his cond ition. H e
d iogram (ECG) and stress test w ere norm al. Urine
d oes not d rink alcohol or u se illicit d rugs, and he
toxicology w as negative. The p atient d enies risk
d enies ad d itional m ed ical history.
factors for heart d isease and d oes not have a fam ily
history of heart d isease.
20. Which of the follow ing ad verse effects is
associated w ith this m ed ication?
17. Which of the follow ing is the m ost likely
d iagnosis? (A) Brad ycard ia
(A) Acu te m yocard ial infarction (B) Galactorrhea
(B) Acu te stress d isord er (C) H yp ertension
(C) Deliriu m (D) Seizu res
(E) Weight loss
Que s tions : 15–27 209

21. Which of the follow ing blood tests w ill requ ire (C) Paranoid p ersonality d isord er
frequ ent, regu lar m onitoring for this p atient? (D) Schizoid p ersonality d isord er
(A) Calciu m level (E) Schizotyp al p ersonality d isord er
(B) Com p lete blood cou nt w ith d ifferential
25. A 28-year-old sep arated w om an is referred
(C) Electrolytes
from her p rim ary care d octor for evalu ation
(D) Thyroid fu nction tests for d ep ression. While she ad m its to occasional
(E) Urinalysis p eriod s of d ysp horia, she claim s to “alw ays
feel em p ty insid e.” Up on fu rther qu estion-
Questions 22 and 23 ing she d em onstrates a p ervasive p attern
of u nstable relationship s, p oor self-im age,
You ad m it an 83-year-old w id ow ed fem ale for
im p u lsiveness, and irritability. Which of the
fu rther evalu ation as she is no longer able to care
follow ing d iagnoses is the m ost ap p rop riate
for herself at hom e. She has lost 30 lb in the p ast
to consid er?
year, has p oor hygiene, and ad m its to increasing
forgetfulness. (A) Antisocial p ersonality d isord er
(B) Bord erline p ersonality d isord er
22. Which of the follow ing tests w ou ld best help (C) Dep end ent p ersonality d isord er
to m ake the correct d iagnosis?
(D) H istrionic p ersonality d isord er
(A) Blessed Rating Scale (E) Schizoid p ersonality d isord er
(B) Folstein Mini-Mental State Exam ination
(MMSE) Questions 26 and 27
(C) Geriatric Rating Scale
A 28-year-old single m an w ith a 10-year history of
(D) Glasgow Com a Scale schizophrenia has been taking his m ed ications reg-
(E) Mental Statu s Exam ination (MSE) u larly. H e now p resents w ith w orsening hallucina-
tions and prom inent thought d isorganization.
23. Which of the follow ing d isord ers w ou ld be
m ost im p ortant to ru le ou t as a cau se of her 26. Which of the follow ing neu rop sychological
clinical p resentation? tests w ou ld best d eterm ine his ability to
(A) Generalized anxiety d isord er organize and correctly p rocess inform ation?
(B) Major d ep ressive d isord er (A) Bend er Gestalt Test
(C) Obsessive-com p u lsive d isord er (B) Draw -A-Person Test
(D) Panic d isord er (C) Lu ria-N ebraska N eu rop sychological
(E) Posttrau m atic stress d isord er Battery
(D) Mini-Mental State Exam ination
24. A 35-year-old patient is encou raged to seek (E) Wisconsin Card Sorting Test (WCST)
“p rofessional help” by his cow orkers. H e
d enies pervasive d epression or anxiety, bu t 27. The above test m easu res fu nctioning at w hich
u pon interview he is od d ly d ressed , expresses of the follow ing lobes of the brain?
u nu sual beliefs and thinking, som e paranoia
regard ing his cow orkers’ m otivations, and (A) Cerebellar
has few close friend s. H e d enies d elu sions (B) Frontal
or hallu cinations, and there is no su icid al or (C) Occip ital
hom icid al id eation. Which of the follow ing (D) Parietal
d iagnoses is the m ost appropriate to consid er? (E) Tem p oral
(A) Avoid ant p ersonality d isord er
(B) N arcissistic p ersonality d isord er
210 7: Pra c tic e Te s t 1

Questions 28 and 29 31. The p atient retu rns 1 m onth later for a follow -
u p exam ination and rep orts th at he exp e-
A 30-year-old w om an w ithout prior p sychiatric his- rienced a gen eralized seizu re. Laboratory
tory is brought to the em ergency d epartm ent by the investigation reveals that the p henytoin level
p olice after being arrested for “breach of the p eace.” is 6.5 m g/ d L (norm al, 10–20 m g/ d L). Which
The w om an w as observed acting irrationally at a of the follow ing is the m ost ap p rop riate
local bu siness w here she d em and ed to sp eak w ith intervention at this tim e?
the presid ent of the com p any claim ing that she had
new id eas for prod u ct d evelopm ent. The patient (A) Ad d a benzod iazep ine.
rep orts that she has not slep t for d ays and that (B) Ad d p henobarbital.
her m ood is “fabulou s.” Urine hum an chorionic (C) Assess com p liance.
gonad otrop in is p ositive. Illicit su bstances w ere not (D) Discontinu e p henytoin and begin
d etected . d ivalproex sod ium .
(E) Increase the p henytoin d ose.
28. Which of the follow ing ad d itional find ings
w ou ld m ost likely be p resent in her history or
32. A 24-year-old w om an w ith a history of
m ental status exam ination (MSE)?
schizophrenia tells you that she w ou ld like
(A) Daytim e sleepiness to becom e p regnant. H er husband has no
(B) Dep ressed affect history of m ental illness. What d o you tell
(C) Racing thou ghts her is the chance of her offsp ring d evelop ing
schizophrenia?
(D) Visu al hallu cinations
(E) Weight loss (A) 1%
(B) 2%
29. Which of the follow ing m ed ical d isord ers can (C) 5%
also present w ith sim ilar sym ptom s? (D) 8%
(A) Cirrhosis (E) 12%
(B) Diabetes m ellitu s
(C) H yp erglycem ia 33. A 28-year-old m arried fem ale p atient is ad m it-
ted to the hospital w ith bizarre behavior and
(D) Rheu m atoid arthritis
d isorganized thinking. Consid eration is given
(E) Thyroid d isord er to a d iagnosis of schizophreniform d isor-
d er. To help w ith the d iagnosis, the patient is
Questions 30 and 31 ad m inistered a test that consists of view ing a
A 24-year-old m an w ith a history of seizu re d is- set of 10 inkblots sequ entially. The exam iner
ord er, alcohol, and cocaine u se d isord er has been scores the patient’s responses to the blots by
incarcerated for assau ltive behavior. The p atient is noting the content of the p ercep tion, the area
evaluated by a neurologist, who prescribes phenytoin of the blot that form s the basis of the response,
(Dilantin). and the asp ects of the area that are u sed to
form the resp onse.
30. Which of the follow ing sid e effects is m ost In w hich of the follow ing classes d oes this
likely associated w ith this m ed ication? projective test belong?

(A) Ebstein anom aly (A) Associations


(B) Gingival hyp erp lasia (B) Choice of ord ering
(C) H ep atic failu re (C) Com p letions
(D) H yp ertension (D) Constru ctions
(E) Leu kocytosis (E) Self-exp ression

You might also like